Location via proxy:   [ UP ]  
[Report a bug]   [Manage cookies]                

Tests Modul 2

Download as docx, pdf, or txt
Download as docx, pdf, or txt
You are on page 1of 99

Students who want to improve the preparation to krok-1 can use websites:

 www.med-test.in.ua
 www.testkrok.org.ua

NEW TESTS MODUL 2*


*correct answer is “A” for all tests

Pathophysiology of the blood system. Anemia caused by blood loss


1. Patient 54 year-old, 5th day after surgical operation. Blood count: erythrocytes 3,6 ∗
1012/l, Hemoglobin 95 g/l, Erythrocyte’s hemoglobin content (color index) 0,78;
Leukocytes 16 ∗ 109/l, Platelets 450 ∗ 109/l Blood picture: anizocytosis, poikilocytosis,
reticulocytes- 3,8%. What anemia does this patient have?
A. Acute posthemorragic anemia
B. Acquired hemolytic anemia
C. Anemia from iron deficiency
D. Hypoplastic anemia
E. Chronic posthemorragic anemia

2. A patient’s blood was analyzed and the decreased erythrocyte’s sedimentation rate
(ESR) was discovered. What disease from the listed below is accompanied with
decreased ESR?
A. Polycythemia
B. Hepatitis
C. Splenomegaly
D. Vitamin B deficiency
E. Myocardial infarction

3. A 38-year-old woman was admitted to the admission-diagnostic department with


uterine bleeding. What are the most likely changes of blood?
Reduction of hematocrit rate
A. Increase of hematocrit rate
B. Leukopenia
C. Leucocytosis
D. Polycythemia

4. A 55-year-old woman consulted a doctor about having continuous cyclic uterine


hemorrhages for a year, weakness, dizziness. Examination revealed skin pallor.
Hemogram: Hb- 70 g/l, erythrocytes - 3, 2*1012/l, color index - 0,6, leukocytes - 6,
0·109/l, reticulocytes - 1%; erythrocyte hypochromia. What anemia is it?
A. Chronic posthemorrhagic anemia
B. Hemolytic anemia
C. Aplastic anemia
D. B12-folate-deficiency anemia
E. Iron-deficiency anemia

1
5. 2 years ago a patient underwent resection of pyloric part of stomach. He complains of
weakness, periodical dark shadows beneath his eyes, dyspnea. In blood: Hb - 70 g/l,
erythrocytes - 3, 0 · 1012/l, colour index - 0,7. What changes of erythrocytes in blood
smears are the most typical for this condition?
A. Microcytes
B. Megalocytes
C. Schizocytes
D. Ovalocytes
E. Macrocytes

6. Packed cell volume of a man was 40% before the trauma. What packed cell volume
will be observed 24 hours after blood loss of 750 ml?
A. 30%
B. 40%
C. 55%
D. 45%
E. 50%

7. A 38-year-old patient with an uterine hemorrhage lasting for 2 days was delivered to
the admission ward. Which of the following will be revealed in the patient’s blood?
A. Decrease in the hematocrit index
B. Eosinophilia
C. Deceleration in ESR
D. Leukocytosis
E. Increase in the colour index

8. A man weighs 80 kg, after long physical activity his circulating blood volume is
reduced down to 5,4 l, hematocrit makes up 50%, whole blood protein is 80 g/l. These
blood characteristics are determined first of all by:
A. Water loss with sweat
B. Increased number of erythrocytes
C. Increased protein concentration in plasm
D. Increased circulating blood volume
E. Increased diuresis

9. Examination of a pregnant woman revealed twice as much concentration of fibrinogen


in blood plasm. What ESR can this woman have?
A. 40-50 mm/h
B. 10-15 mm/h
C. 2-12 mm/h
D. 5-10 mm/h
E. 0-5 mm/h

10. Blood count of an athlete is as follows: erythrocytes - 5, 5 · 1012/l, Hb- 180 g/l,
leukocytes - 7 · 109/l, neutrophils - 64%, basophils - 0,5%, eosinophils - 0,5%,
monocytes - 8%, lymphocytes - 27%. First of all, such results indicate the stimulation of:
A. Erythropoiesis
B. Leukopoiesis
2
C. Lymphopoiesis
D. Granulocytopoiesis
E. Immunogenesis

11. A 42 year old patient complains of pain in the epigastral area, vomiting; vomit masses
have the colour of "coffee-grounds", the patient has also melena. Anamnesis records
gastric ulcer. Blood formula: erythrocytes - 2, 8*1012/l, leukocytes - 8 · 109/l, Hb- 90
g/l. What complication is it?
A. Hemorrhage
B. Penetration
C. Perforation
D. Canceration
E. Pyloric stenosis

12. A 26 year old pregnant woman is under treatment at an in-patient hospital. After a
continuous attack of vomiting she was found to have reduced volume of circulating
blood. What kind of change in general blood volume is the case?
A. Polycythemic hypovolemia
B. Simple hypovolemia
C. Oligocythemic hypovolemia
D. Polycythemic hypervolemia
E. Oligocythemic hypervolemia

13. On the fifth day after the acute blood loss a patient has been diagnosed with
hypochromic anemia. What is the main mechanism of hypochromia development?
A. Release of immature red blood cells from the bone marrow
B. Impaired iron absorption in the intestines
C. Increased destruction of red blood cells in the spleen
D. Impaired globin synthesis
E. Increased excretion of body iron

14. In a car accident a man got injured and lost a lot of blood. What changes in peripheral
blood are most likely to occur on the 2th day after the injury?
A. Erythropenia
B. Hypochromia
C. Anisocytosis
D. Microplania
E. Significant reticulocytosis

15. Clinical examination of residents of mountainous terrain, located at an altitude of


3,000 meters showed an increased amount of red blood cells. The reason for this is:
A. increasing production of erythropoietin;
B. The change in the function of the spleen;
C. The increase in blood volume;
D. The increase in the synthesis of vitamin B12;
E. hemoconcentration

3
16. A man of 40 years, complains of fatigue, weakness. In recent history there was a
trauma with blood loss. OBJECTIVE: pale skin and visible mucous membranes, pulse 90
beats. / Min. What sign will be obligatory for anemia, in the patient?
A. decrease in hemoglobin concentration
B. The decrease in blood volume;
C. The reduction of the color indicator;
D. increase of reticulocytes;
E. resizing of erythrocytes.

17. In patient 37 years in a blood test: Hb - 60 g / l, erythrocytes - 3,0*1012 / L CPU -


0.6. Wbc unchanged, platelets - 200 *109 / L, reticulocytes 30%, ESR - 18 mm / hour.
Anisocytosis, poikilocytosis. What is the mechanism of anemia?
A. acute posthemorrhagic;
B. hemolytic;
C. iron deficiency;
D. hypoplastic;
E. B12 folic deficiency.

18. A man lives for a long time at high altitudes. What changes in the blood and
circulatory system will he get?
A. An increase in hemoglobin;
B. The increase in the number of white blood cells;
C. The increase in the diameter of blood vessels;
D. The reduction in the number of white blood cells;
E. decrease in frequency of the pulse.

19. The patient 32 years old was taken to the clinic suffering from massive blood loss due
to road trauma. Pulse 110 beats. / Min., Respiratory rate - 22 in 1 min, blood pressure,
100/60 mm Hg. Art. What change in blood among listed will be the most typical in 1
hour after blood loss?
A. hypovolemia;
B. eritropeniya;
C. hypochromia of erythrocytes;
D. leukopenia;
E. hypoproteinemia.

20. In a woman after childbirth complications, which were accompanied by significant


blood loss, was diagnosed hypochromic anemia. What pathological forms of erythrocytes
are specific to this anemia?
A. anulocytes;
B. macrocytes;
C. sickle-shaped erythrocytes;
D. spherocytes;
E. microcytes.

21. In a woman 46 years old on a background of moderate menorrhagia developed


anemia: Er - 3,6h1012 / l, HB - 60 g / l, cs - 0.5 reticulocytes - 0.1%. In a smear:

4
hypochromia, anulocytosis, microcythemia, poikilocytosis. What type of anemia
developed in a patient?
A. iron deficiency.
B. acute posthemorrhagic;
C. hemolytic;
D. aplastic;
E. B12 folic deficiency;

22. Under a planned examination of the teenagers living in the highlands, there was an
increase in the number of red blood cells and hemoglobin in peripheral blood. What
caused noted polycythemia?
A. exogenous hypoxia;
B. bronchopulmonary diseases;
C. congenital heart disease;
D. blood clots due to the large loss of body fluids;
E. Vakeza’s disease.

23. In cases of anemia there are degenerative and regenerative forms of erythrocytes in
peripheral blood . Name the regenerative forms of erythrocytes.
A. polychromatic erythrocytes.
B. microcytes;
C. spherocytes;
D. poikilocytes;
E. hyperchromic red blood cells;

24. On the fifth day after acute blood loss in the patient revealed a hypochromic anemia
of erythrocytes. What is the main mechanism in the development of this process?
A. inflow of immature erythrocytes from bone marrow;
B. impaired iron absorption in the intestine;
C. increased destruction of erythrocytes in the spleen;
D. violation of globin synthesis;
E. the increase of iron release from the body.

25. Due to the injury patient has lost 25% of circulating blood volume. Name the urgent
mechanism of compensation of blood loss.
A. interstitial fluid inflow in the vessels;
B. renewal of blood protein;
C. The increase in the number of reticulocytes;
D. The recovery of the number of erythrocytes;
E. the activation of erythropoiesis.

26. A child who received thermal burns up to 40% of the body surface resulting to
careless handling of fire, the hematocrit reveals a violation of plasma and formed
elements ratio. What form of violation of the total blood volume is observed at the same
time?
A. polycythemic hypovolemia.
B. polycythemic hypervolemia;
C. oligocythemic hypovolemia;
5
D. normocythemic hypovolemia;
E. oligocythemic hypervolemia;

27. In the patient 42 years old complaints of pain in the epigastric region, vomiting;
vomit-colored "coffee grounds". In the anamnesis - stomach ulcer. Blood: erythrocytes -
2.8 h1012 / l leukocytes - 8h109 / L, hemoglobin 90 g / L. Select the most likely
complication arose in the patient?
A. bleeding;
B. penetration;
C. perforations;
D. degeneration in cancer;
E. pyloric stenosis.

28. In the general survey of the patient revealed hyperemia of the skin with cyanotic hue.
Attention is drawn to block of the patient and slowing of his movements. Blood test
showed that erythrocytes 9*1012 / L, hematocrit 60%. At what pathological condition
will be an absolute erythrocytosis?
A. Vaquez’s disease;
B. megaloblastic anemia;
C. lymphoma;
D. hemodilution;
E. –

29. A man 57 years old after the survey was diagnosed B12 deficiency anemia,
prescribed treatment. After 3 days was made a controlling blood test. Which of the
following criteria is an indication increasing erythropoiesis?
A. increasing the number of reticulocytes;
B. increase of the level of hemoglobin;
C. reduction of the color indicator;
D. normoblastic type of hematopoiesis;
E. increase the number of white blood cells.

30. In a patient with hypochromic anemia there is a whipped hair fall, there is increased
fragility of nails and taste disturbance. What is the mechanism of these symptoms?
A. ferriferous enzymes deficiency.
B. vitamin B12 deficiency;
C. the decrease in production of parathyrin;
D. vitamin A deficiency;
E. the decrease in the production of thyroid hormones;

31. In the blood of a 26-year-old man 18% of erythrocytes of the spherical, ballshaped,
flat and thorn-like shape have been revealed. Other eritrocytes were in the form of the
concavo-concave disks. How is this phenomenon called?
A. Physiological poikilocytosis
B. Pathological poikilocytosis
C. Physiological anisocytosis
D. Pathological anisocytosis
E. Erytrocytosis
6
32. Due to trauma the patient has lost 25% of circulating blood volume. Name the
emergency compensatory mechanism against blood loss:
A. Interstitial fluid inflow to the vessels
B. Restoration of blood protein composition
C. Increase of reticulocyte number
D. Restoration of erythrocyte number
E. Erythropoiesis activation

33. A 30-year-old patient’s blood test revealed the following: erythrocyte count is 6 ·
1012/l, hemoglobin is 10,55 mmol/l. Vaquez’s disease was diagnosed. Name the
leadingpart of pathogenesis in this case:
A. Neoplastic erythroid hyperplasia
B. Iron-deficiency
C. B12-deficiency
D. Hypoxia
E. Acidosis

34. Blood test of an athlete shows the following: erythrocytes - 5,5 · 1012/l, hemoglobin -
180 g/l, leukocytes - 7 · 109/l, neutrophils - 64%, basophils - 0,5%, eosinophils - 0,5%,
monocytes - 8%, lymphocytes - 27%. These values primarily indicate the stimulationof:
A. Erythropoiesis
B. Leukopoiesis
C. Lymphopoiesis
D. Granulocytopoiesis
E. Immunogenesis

35. 10 minutes after the beginning of heavy physical work a person demonstrates
increase of erythrocyte number in blood from 4,0·1012/l to 4,5·1012/l. What is the cause
of this phenomenon?
A. Erythrocytes exit from depot
B. Suppression of erythrocyte destruction
C. Erythropoiesis activation
D. Increase of cardiac output
E. Water loss

Hemolytic anemia and dyserythropoietic anemia


1. A patient is diagnosed with iron deficiency sideroahrestical anemia, course of which is
accompanied by hyperpigmentation of the skin, the development of pigment liver
cirrhosis, damage of the pancreas and heart. The iron content in the serum was increased.
What violation of iron metabolism caused this disease?
A. disuse of iron and its deposition in tissues;
B. an excessive intake of iron from food;
C. the impaired iron absorption in the intestine;
D. increased use of iron by the body;
E. a combined deficiency of vitamin B12 and iron.

7
2. Substitution of the glutamic acid on valine was revealed while examining initial
molecular structure. For what inherited pathology is this symptom typical?
A. Sickle-cell anemia
B. Thalassemia
C. Minkowsky-Shauffard disease
D. Favism
E. Hemoglobinosis

3. Examination of a 43 y.o. anephric patient revealed anemia symptoms. What is the


cause of these symptoms?
A. Reduced synthesis of erythropoietins
B. Enhanced destruction of erythrocytes
C. Iron deficit
D. Vitamin B12 deficit
E. Folic acid deficit

4. A 56 year old patient came to a hospital with complaints about general weakness,
tongue pain and burning, sensation of limb numbness. In the past he underwent resection
of forestomach. In blood: Hb- 80 g/l; erythrocytes - 2, 0*1012/l; colour index - 1,2,
leukocytes - 3, 5* 109/l. What anemia type is it?
A. B12-folate deficient
B. Hemolytic
C. Posthemorrhagic
D. Aplastic
E. Iron-deficient

5. A patient was diagnosed with autoimmune hemolitic cytotoxic anemia. What


substances are antigens in II type allergic reactions?
A. Modified receptors of cell membranes
B. Antibiotics
C. Hormones
D. Serum proteins
E. Inflammation modulators

6. As a result of increased permeability of the erythrocyte membrane in a patient with


microspherocytic anaemia (Minkowsky-Shauffard disease) cells receive sodium ions and
water. Erythrocytes take form of spherocytes and can be easily broken down. What is the
leading mechanism of erythrocyte damage in this case?
A. Electrolytic osmotic
B. Calcium
C. Acidotic
D. Protein
E. Nucleic

7. A 34 year old woman was diagnosed with hereditary microspherocytic hemolytic


anemia (Minkowsky-Shauffard disease). What mechanism caused hemolysis of
erythrocytes?
A. Membranopathy
8
B. Enzymopathy
C. Hemoglobinopathy
D. Autoimmune disorder
E. Bone marrow hypoploasia

8. A 25 year old Palestinian woman complains of weakness, dizziness, dyspnea. In


anamnesis: periodically exacerbating anemia. In blood: Hb - 60 g/l, erythrocytes - 2,
5*1012/l, reticulocytes – 35%, anisocytosis and poikilocytosis of erythrocytes, a lot of
target cells and polychromatophils. What type of anemia is it?
A. Thalassemia
B. Sickle-cell anemia
C. Minkowsky-Shauffard disease
D. Addison-Biermer disease
E. Glucose 6-phosphate dehydrogenasedeficient anemia

9. Blood test of a patient suffering from atrophic gastritis gave the following results:
RBCs - 2, 0 * 1012/l, Hb- 87 g/l, colour index - 1,3, WBCs - 4, 0 *109/l, thrombocytes -
180 * 109/l. Anaemia migh have been caused by the following substance deficiency:
A. Vitamin B12
B. Vitamin A
C. Vitamin K
D. Iron
E. Zinc

10. Examination of a 52-year-old female patient has revealed a decrease in the amount of
red blood cells and an increase in free hemoglobin in the blood plasma
(hemoglobinemia). Color index is 0,85. What type of anemia is being observed in the
patient?
A. Acquired hemolytic
B. Hereditary hemolytic
C. Acute hemorrhagic
D. Chronic hemorrhagic
E. Anemia due to diminished erythropoiesis

11. Degenerative changes in posterior and lateral columns of spinal cord (funicular
myelosis) caused by methylmalonic acid accumulation occur in patients with B12-
deficiency anemia. This results in synthesis disruption of the following substance:
A. Myelin
B. Acetylcholine
C. Norepinephrine
D. Dopamine
E. Serotonin

12. A blood drop has been put into a test tube with 0,3% solution of NaCl. What will
happen to erythrocytes?
A. Osmotic hemolysis
B. Shrinkage
C. Mechanical hemolysis
9
D. Any changes will be observed
E. Biological hemolysis

13. A man 40 years old was admitted to the hospital after been bitten by a snake. Where
more likely will be hemolysis in this case?
A. in the bloodstream;
B. in liver cells;
C. in spleen cells;
D. In bone marrow;
E. in renal parenchyma.

14. A woman 60 years old complains of severe weakness, shortness of breath and
palpitations with little physical exertion, fatigue and a feeling of "pins and needles" on
the extremities. For many years, suffering from atrophic gastritis. Determined by the
pallor of skin and mucous membranes, tongue is crimson. Blood test: Hb -60 g / l,
erythrocytes - 1.5 * 1012 / L, color index - 1.2, a significant anisocytosis of erythrocytes
isolated macrocytes. What anemia most likely occurs in the patient?
A. B12 - folic acid deficiency anemia;
B. chronic hemorrhagic anemia;
C. acquired hemolytic anemia;
D. Iron deficiency anemia;
E. Iron refrectary anemia.

15. In the neonate 3 weeks old (the parents are healthy father-Rh "+", mother-Rh "-"),
clear jaundice of the skin and visible mucous. In the blood erythrocytes 3,7-1012 / l, Hb-
105 g / l, elevated indirect bilirubin of blood. What is this anemia by pathogenesis?
A. acquired hemolytic;
B. hemorrhagic;
C. hereditary hemolytic;
D. hypoplastic;
E. B12 folic deficiency.

16. Woman 34 years old complains of general weakness, jaundice of the skin.
Objectively: skin and visible mucous membranes icteric, liver and spleen are enlarged.
Blood test: Hb - 80 g / l, erythrocytes - 3.0 * 1012 / L, color index - 0.8; reticulocytes -
1.8%, ESR - 19 mm / h, microspherocytosis, hyperbilirubinemia (bilirubin free); in urine
- urobilinuria. Was diagnosed a hereditary microspherocytic hemolytic anemia
( Minkowski-Chauffard disease). Which of the following mechanisms caused hemolysis
of erythrocytes in a patient?
A. membranopaty;
B. hemoglobinopathies;
C. enzimopaty;
D. autoimmunity;
E. The bone marrow hypoplasia.

17. The patient 36 years old complains of general weakness, bleeding of gums, nasal
bleedings . Symptoms appeared after undergoing respiratory viral infection, in the
treatment were used sulfonamides. Liver, spleen, lymph nodes were not enlarged. In the
10
blood: hyporegenaratory normochromic anemia, leukopenia, thrombocytopenia. In the
bone marrow: a decrease in the number of mielocariocytes. What is the anemia by
mechanism of development?
A. hypoplastic;
B. posthemorrhagic;
C. hemolitic;
D. iron deficiency;
E. B12 folic deficiency.

18. In blood analysis of 35-year-old patient: Hb - 58 g / l erythrocytes - 1.3 * 1012 / l, the


CI - 1.3, leukocytes - 2.8 *109 / L, platelets - 110 x 109 / l , 0.2% reticulocytes, ESR - 35
mm / hour. Determined polysegmented neutrophils and also Jolly corpuscles and Kebot
rings. What is the mechanism of anemia?
A. B12 folic deficiency
B. hemorrhagic;
C. hemolytic;
D. iron deficiency;
E. hypoplastic.

19. In the woman of 40 years revealed a hemolitic anemia caused by a genetic defect in
the enzyme glucose-6-phosphate dehydrogenase in erythrocytes. Formation of what
substance of pentose phosphate pathway is mostly violated in this case?
A. NADPH
B. dioxyacetonphosphate;
C. phosphoenolpyruvate;
D. FADN2;
E. glucose-6-phosphate.

20. The patient was admitted to the clinic with complaints of weakness, dyspnea, fatigue,
dizziness. In the blood: er. - 1,8~10*12 / l; Hb - 80 g / l; CI - 1.33; Leuk. - 3,2~10*9 / l.
In a smear: anisocytosis, poikilocytosis, megaloblasts, megalocytes. What is the most
possible diagnosis?
A. B12-deficiency anemia;
B. posthemorrhagic anemia;
C. acute leukemia;
D. Iron deficiency anemia;
E. immunohemolytic anemia.

21. In a patient, carrier of genetic sickle anomaly of erythrocytes, the incidence of


pneumonia was accompanied by hemolytic crisis and development of anemia. What is
the direct cause of hemolytic crisis in this case?
A. hypoxia caused by pneumonia
B. hyperoxia;
C. heterozygosity for Hb S;
D. mutation of the structural gene;
E. changes in osmolarity of blood.

11
22. The patient has a mutated gene responsible for the synthesis of hemoglobin. This led
to the development of the disease - sickle cell anemia. How is called a pathological
hemoglobin, which is revealed in this disease?
A. HbS
B. HbA;
C. HbF;
D. HbA1;
E. Bart-Hb.

23. In the patient there is a significant reduce in content of hemoglobin and


erythrocytes in blood from time to time. From the anamnesis revealed that these attacks
always occur after consumption of faba bean. What form of anemia is in this patient?
A. fermentopathy
B. membranopathy;
C. Iron deficiency anemia;
D. hemoglobinopathy;
E. obtained hemolytic anemia.

24. In a man of 40 years old was diagnosed a sickle cell anemia. What mechanism causes
reduction of red blood cells of the patient?
A. intracellular hemolysis;
B. intravascular hemolysis;
C. the lack of iron in the body;
D. the lack of vitamin B12 and folic acid;
E. chronic blood loss.

25. In a Patient I., aged 13, was revealed a fermentopathy. What is the leading
mechanism of hemolysis at deficiency of glucose-6-phosphate dehydrogenase?
A. decrease in antioxidant protection
B. The energy deficit;
C. hypoxia;
D. the violation of protein conformation;
E. the reduction of electric potential.

26. In a patient with hypochromic anemia in erythrocytes there is 45% HbS and 55% Hb
A1. What form of anemia is in patient?
A. sickle cell anemia;
B. Alpha-thalassemia;
C. the Addison-Birmer disease;
D. the Glucose-6- phosphate dehydrogenase anemia;
E. the microspherocytic anemia.

27. In a patient, who had arrived from Tunisia, was revealed alpha-thalassemia with
hemolysis of erythrocytes and jaundice. The disease was diagnosed on the basis of the
presence in the blood of:
A. targetlike erythrocytes
B. granular erythrocytes;
C. polychromatic erythrocytes;
12
D. normocytes;
E. reticulocytes.

28. In a patient with hemolytic jaundice in the blood smear erythrocytes are present in
the form of microspherocytes 1 - 6 in sight. What is the possible reason of hemolysis of
erythrocytes, causing the emergence of this form of jaundice?
A. hereditary defect of their membranes
B. hereditary defect in structure of hemoglobin;
C. violation of enzyme systems of erythrocyte;
D. 4the impacts of bile acids on the erythrocyte's membrane ;
E. bilirubin effects on erythrocyte's membrane.

29. In a patient after gastrectomy developed a B12 folic acid deficiency anemia . Which
of the following color indicators is typical for this disease?
A. 1.4;
B. 1,0;
C. 0.8;
D. 0,5;
E. 0.2.

30. In a patient of 35 years old developed an immune microspherocytic hemolytic


anemia. What rate of serum will mostly increase?
A. indirect bilirubin;
B. direct bilirubin;
C. stercobilinоgene;
D. mezobilinogene;
E. protoporphyrin.

31. A patient with hypochromic anemia has splitting and loss of hair, increased nail
brittling and taste alteration. What is the mechanism of the symptoms development?
A. Deficiency of iron-containing enzymes
B. Deficiency of vitamin B 12
C. Decreased production of parathyrin
D. Deficiency of vitamin А
E. Decreased production of thyroid hormones

32. A 15 year old girl has pale skin, glossitis, and gingivitis. Blood count: erythrocytes -
3, 3 * 1012/l, hemoglobin - 70 g/l, colour index - 0,5. Examination of blood smear
revealed hypochromia, microcytosis, and poikilocytosis. What type of anemia is it?
A. Iron-deficient
B. B12-folic acid-deficient
C. Sickle-cell
D. Hemolytic
E. Thalassemia

33. A 37-year-old female patient complains of headache, vertigo, troubled sleep,


numbness of limbs. For the last 6 years she has been working at the gas-discharge lamp-
producing factory in the lead-processing shop. Blood test findings: low hemoglobin and
13
RBC level, serum iron concentration exceeds the norm by several times. Specify the type
of anemia:
A. Iron refractory anemia
B. Iron-deficiency anemia
C. Minkowsky-Shauffard disease
D. Hypoplastic anemia
E. Metaplastic anemia

34. A patient is diagnosed with irondeficiency sideroachrestic anemia, progression of


which is characterized by skin hyperpigmentation, pigmentary cirrhosis, heart and
pancreas affection. Iron level in the blood serum is increased. What disorder of iron
metabolism causes this disease?
A. Failure to assimilate iron leading to iron accumulation in tissues
B. Excessive iron intake with food
C. Disorder of iron absorption in bowels
D. Increased iron assimilation by body
E. –

35. Male 47 years old complains of severe weakness, dizziness. Six months ago,
underwent surgery for gastric resection Billroth-II method over the perforation of
stomach ulcers. On examination: reduced power (height 172 cm, weight 60 kg), skin and
mucous membranes are pale, skin is dry, scaly. Blood test: 80 g Hb / l, erythrocytes - 3.5
1012 / l, the CI - 0.69 ESR - 15 mm / hour. Serum iron - 5.4 mmol / l. 4. Hypochromia of
erythrocytes. What anemia most likely occurs in a patient?
A. iron deficiency ;
B. chronic hemorrhagic;
C. hemolytic;
D. B12 folic acid;
E. ironrefractory.

36. In the 6 month of pregnancy woman got an expressed iron-deficiency anemia.


Diagnostic sign of it was the appearance in the blood of:
A. anulocytes;
B. microcytes;
C. poikilocytes;
D. reticulocytes;
E. normocytes.

37. In the hematological department was admitted a woman 20 years old with complaints
of general weakness, fatigue, drowsiness, periodic faintings. In the analysis of blood: red
blood cells - 3,0*1012 / l, hemoglobin - 74 g / l, cs - 0.74, anisocytosis, poikilocytosis,
anulocytosis, serum iron increased (37 mmol / l). What is the most probable mechanism
of anemia development?
A. violation of the synthesis of porphyrins;
B. chronic blood loss;
C. iron deficiency;
D. intravascular hemolysis;
E. cyanocobalamin deficiency.
14
38. Female 37 years old complains of general weakness, dizziness, difficulty in
swallowing food, the desire to eat chalk. Objective: t = 36,50C, BH 20 min., Pulse 96 per
minute. Blood pressure 110/70 mm Hg. Art. Skin and visible mucous membranes are
pale. In the blood: Hb - 70 g / l, er. - 3.0 T / L, CI - 0.7, Leuk. - 4.7 T / l, reticulocytes -
2%, EOZ. - 2%, 64% segm, lymph. - 26%, mon. - 8%. ESR - 15 mm / h. Serum iron - 7.3
mmol / l. Deficiency of what factor led to the emergence of disease?
A. Iron;
B. protein;
C. folic acid;
D. Vitamin B6;
E. Vitamin B12;

39. In a child on artificial feeding by cow's milk, developed a severe anemia: red blood
cells - 4 *1012 / l, Hb - 68 g / L, reticulocytes - 0%. What anemia developed in a child?
A. iron deficiency;
B. congenital hemolytic;
C. B12 deficiency;
D. sickle cell;
E. hypoplastic.

40. The patient underwent operation concerning resection of the pyloric stomach. A year
later, complains of weakness, the periodic appearance of dark circles under the eyes,
shortness of breath. In the blood: er. - 3,0*1012 / l; Hb - 70 g / l. What changes in the
erythrocytes in peripheral blood smears are typical for this disease?
A. hypochromic erythrocytes;
B. hyperchromic erythrocytes;
C. erythrocytes with Kebo corpuscles;
D. erythrocytes with Jolly corpuscles;
E. macrocytes.

41. A year after subtotal gastrectomy for ulcer of the lesser curvature revealed changes in
the laboratory analysis of blood - anemia, leukopenia and thrombocytopenia, CI1.3, there
is a presence of megaloblasts and megalocytes. Deficiency of what factor leads to these
changes?
A. castle factor;
B. hydrochloric acid;
C. the mucin;
D. pepsin;
E. gastrin.

42. In a woman of 40 years revealed atrophic and inflammatory processes in the oral
cavity, the violation of deep sensitivity. Was diagnosed Addison-Biermer anemia. What
is the type of anemia in a patient?
A. megaloblastic;
B. regenerative;
C. normochromic;
D. hemolytic;
15
E. iron deficiency.

43. In the patient's blood is observed the following: erythrocytes - 3.0*1012 / L; Hb - 90g
/ l; reticulocytes - 0.5%. In a smear: poikilocytes, hypochromic red blood cells. Serum
iron – 80 mkmol / l. For what pathology is it typical?
A. ironrefractory anemia;
B. Minkowski-Chauffard disease;
C. Iron deficiency anemia;
D. B12 deficiency anemia;
E. sickle-cell anemia.

44. In the process of hemoglobin catabolism iron is released and then as a part of special
transport protein is returned to the bone marrow, to be used again for hemoglobin
synthesis. Name this transport protein:
A. Transferrin
B. Transcobalamin
C. Haptoglobin
D. Ceruloplasmin
E. Albumin

45. A patient with hypochromic anemia has hair with split ends and suffers from hair
loss. The nails are brittle. Gustatory sensations are affected. What is the mechanism of
development of these symptoms?
A. Iron enzymes deficiency
B. Vitamin B12 deficiency
C. Low production of parathyroid hormone
D. Vitamin A deficiency
E. Low production of thyroid hormones

46. One of the causes of pernicious anemia is disturbed synthesis of transcorrin - Castle’s
intrinsic factor - by the parietal cells of the stomach. What substance is called Castle’s
extrinsic factor?
A. Cobalamin
B. Folic acid
C. Pyridoxine
D. Riboflavin
E. Biotin

47. A patient with glossitis presents with disappearance of lingual papillae, reddening and
burning pain in the tongue. Blood test:
erythrocytes - 2.2 1012/l, hemoglobin - 103 g/l, color index - ·1.4. What type of anemia is
it?
A. B12 folate-deficiency
B. Iron deficiency
C. α-thalassemia
D. β-thalassemia
E. Iron refractory

16
48. A patient has been diagnosed with severe B12-deficient anemia with hemopoiesis
disturbance. Anamnesis states total gastrectomy. What cells allow to confirm this
diagnosis, if they are present in the peripheral blood?
A. Megalocytes
B. Microcytes
C. Ovalocytes
D. Normocytes
E. Anulocytes
49. A patient with glossitis presents with disappearance of lingual papillae, reddening and
burning pain in the tongue. Blood test: erythrocytes 2,2 · 10*12/l, hemoglobin - 103 g/l,
color index - 1,4. What type of anemia is it?
A. B12 folate-deficient
B. Iron deficiency
C. α-thalassemia
D. β-thalassemia
E. Iron refractory

50. Examination of a 52-year-old woman has revealed a decrease in the amount of red
blood cells and an increase in free hemoglobin in the blood plasma (hemoglobinemia).
Color index is 0,85. What type of anemia is being observed in the patient?
A. Acquired hemolytic
B. Hereditaryhemolytic
C. Acute hemorrhagic
D. Chronic hemorrhagic
E. Anemia due to diminished erythropoiesis

51. A patient has been diagnosed with severe B12-deficient anemia with hemopoiesis.
Anamnesis states total gastrectomy.What cells allow to confirm this diagnosis, if they are
absent in the peripheral blood?
A. Megalocytes
B. Microcytes
C. Ovalocytes
D. Normocytes
E.Anulocytes

Pathology of leukocytes

1. A 16-year-old boy was performed an appendectomy. He has been hospitalized for right
lower quadrant abdominal pain within 18 hours. The surgical specimen is edematous and
erythematous. Infiltration by what of the following cells is the most typical for the
process occuring here?
A. Neutrophils
B. Eosinophils
C. Basophils
D. Limphocytes
E. Monocytes

17
2. Blood sampling for bulk analysis is recommended to be performed on an empty
stomack and in the morning. What changes in blood composition can occur if to perform
blood sampling after food intake?
A. Increased contents of leukocytes
B. Increased contents of erythrocytes
C. Increased plasma proteins
D. Reduced contents of thrombocytes
E. Reduced contents of erythrocytes

3. A patient operated on complicated appendicitis has the following changes of blood


count: erythrocytes - 4, 0*1012/l, Нb - 120 g/l, color index - 0,9, leukocytes – 18*109/l,
basophils - 0, eosinophils - 0, myelocytes - 0, juvenile - 0, stab neutrophils - 20,
segmentonuclear neutrophils - 53, lymphocytes - 21, monocytes - 5. How is such nuclear
shift of leukocyte formula called?
A. Degenerative left shift
B. Right shift
C. Regenerative left shift
D. Hyperregenerative
E. Regeneratively-degenerative

4. 24 hours after appendectomy blood of a patient presents neutrophilic leukocytosis with


regenerative shift. What is the most probable mechanism of leukocytosis development?
A. Amplification of leukopoiesis
B. Redistribution of leukocytes in the organism
C. Decelerated leukocyte destruction
D. Deceleratied emigration of leukocytes to the tissues
E. Amplification of leukopoiesis and decelerated emigration of leukocytes to the
tissues

5. A 23 y.o. patient complains of weakness, temperature rise up to 38 − 400C.


Objectively: liver and spleen are enlarged. Hemogram: Hb- 100 g/l, erythrocytes - 2, 9
*1012/l, leukocytes - 4, 4*109/l, thrombocytes – 48*109/l, segmentonuclear neutrophils -
17%, lymphocytes - 15%, blast cells - 68%. All cytochemical reactions are negative.
Make a hematological conclusion:
A. Undifferentiated leukosis
B. Chronic myeloleukosis
C. Acute myeloblastic leukosis
D. Acute lymphoblastic leukosis
E. Acute erythromyelosis

6. Two hours after an exam a student had a blood count done and it was revealed that he
had leukocytosis without significant leukogram modifications. What is the most probable
mechanism of leukocytosis development?
A. Redistribution of leukocytes in the organism
B. Leukopoiesis intensification
C. Deceleration of leukocyte lysis
D. Deceleration of leukocyte migration to the tissues
E. Leukopoiesis intensification and deceleration of leukocyte lysis
18
7. Parents of a 3 year old child have been giving him antibiotics with purpose of
preventing enteric infections for a long time. A month later the child’s condition changed
for the worse. Blood examination revealed apparent leukopenia and granulocytopenia.
What is the most probable mechanism of blood changes?
A. Myelotoxic
B. Autoimmune
C. Redistributive
D. Age-specific
E. Hemolytic

8. A 26 year old man is in the torpid shock phase as a result of a car accident. In blood: 3,
2*109/l. What is the leading mechanism of leukopenia development?
A. Redistribution of leukocytes in bloodstream
B. Leukopoiesis inhibition
C. Disturbed going out of mature leukocytes from the marrow into the blood
D. Lysis of leukocytes in the blood-forming organs
E. Intensified elimination of leukocytes from the organism

9. Examination of a patient admitted to the surgical department with symptoms of acute


appendicitis revealed the following changes in the white blood cells: the total count of
leukocytes is 16 * 109/l. Leukocyte formula: basophils - 0, eosinophils - 2%, juvenile
forms - 2%, stabnuclear - 8%, segmentonuclear - 59%, lymphocytes - 25%, monocytes-
4%. The described changes can be classified as:
A. Neutrophilia with regenerative left shift
B. Neutrophilia with right shift
C. Neutrophilia with degenerative left shift
D. Neutrophilic leukemoid reaction
E. Neutrophilia with hyperregenerative left shift

10. A 5 year old child is ill with measles. Blood analysis revealed increase of total
number of leukocytes up to 13*109/l. Leukogram: basophils - 0, eosinophils - 1,
myelocytes - 0, juvenile neutrophils - 0, band neutrophils - 2, segmented neutrophils - 41,
lymphocytes - 28, monocytes - 28. Name this phenomenon:
A. Monocytosis
B. Agranulocytosis
C. Lymphocytosis
D. Eosinopenia
E. Neutropenia

11. A patient complains about dyspnea provoked by the physical activity. Clinical
examination revealed anemia and presence of the paraprotein in the zone of gamma-
globulins. To confirm the myeloma diagnosis it is necessary to determine the following
index in the patient’s urine:
A. Bence Jones protein
B. Bilirubin
C. Haemoglobin
D. Ceruloplasmin
19
E. Antitrypsin

12. Having helped to eliminate consequences of a failure at a nuclear power plant, a


worker got an irradiation doze of 500 roentgen. He complains of headache, nausea,
dizziness. What changes in leukocytes quantity can be expected 10 hours after
irradiation?
A. Neutrophilic leukocytosis
B. Lymphocytosis
C. Leukopenia
D. Agranulocytosis
E. Leukemia

13. A patient suffering from chronic myeloleukemia has got the following symptoms of
anemia: decreased number of erythrocytes and low hemoglobin concentration, oxyphilic
and polychromatophilic normocytes, microcytes. What is the leading pathogenetic
mechanism of anemia development?
A. Substitution of haemoblast
B. Intravascular hemolysis of erythrocytes
C. Deficiency of vitamin B12
D. Reduced synthesis of erythropoietin
E. Chronic hemorrhage

14. A 3-year-old child had eaten some strawberries. Soon he developed a rash and
itching. What was found in the child’s leukogram?
A. Eosinophilia
B. Hypolymphemia
C. Neutrophilic leukocytosis
D. Monocytosis
E. Lymphocytosis

15. A man 27 years old is diagnosed with acute myelogenous leukemia. Despite ongoing
therapy has an amplifing anemia. What is this anemia by pathogenesis?
A. hypoplastic
B. hemorrhagic;
C. hemolytic;
D. iron deficiency;
E. Vitamin B12 (folic-) deficiency;

16. The patient, 59 years old, hospitalized in the surgical department with an exacerbation
of chronic osteomyelitis of the left tibia. In the analysis of blood: leukocytes - 15,0*109 /
l; WBC: myelocytes - 2% metamyelocytes - 8% stab - 28% segmented neutrophiles -
30% lymphocytes - 29% monocytes - 3%. What type of nucleus shift of neutrophils is in
a patient?
A. regenerative shift to the left;
B. degenerative shift;
C. the regenerative-degenerative shift;
D. hyperregenerative shift;
E. shift to the right.
20
17. A woman of 35 years was admitted to the the surgical department. Diagnosis:
appendicitis?, local peritonitis? In the analysis of blood: leukocytes – 23*109 / l;
neutrophils - 77%, including Miel. - 2%; th. - 8%, fell. - 12%; segm.-55%. Yixian - 0.40.
What is a type of nucleus shift of neutrophils?
A. regenerative shift to the left;
B. regenerative-degenerative shift to the left;
C. the degenerative shift to the left;
D. hyperregenerative shift to the left;
E. shift to the right.

18. A man 38 years old, liquidator of the accident at the NPP. Objectively: pale skin and
visible mucous membranes, respiratory rate - 22 in 1 min, pulse - 95 in 1 min. In the
blood erythrocytes - 2,4 *1012 / L, hemoglobin - 70 g / l, leukocytes - 3,3*109 / L,
platelets – 130*109 / l. How to assess changes in the blood correctly?
A. pancytopenia.
B. anemia;
C. eukopenia;
D. thrombocytopenia;
E. eritropeniya;

19. Male 38 years, has been working for 12 years at the chemical plant in the workshop
of benzene production. At medical examinations in his blood was revealed: WBC-
3.0*109 / L, neutrophils - 42% (M.-0-yun. 0 pal. 13% segm.-29%), neutrophils
hypersegmentation, Yixian = 0.42. Which of the following shifts of leukocyte formula is
in men?
A. degenerative to the left
B. to the left regenerative;
C. to the left regenerative-degenerative;
D. to the left hyperregenerative;
E. shift to the right.

20. The patient of 65 years old was diagnosed with a myocardial infarction. In blood
analysis there is a neutrophil leukocytosis with a left shift. What factors could determine
such a phenomenon?
A. decomposition products of tissues;
B. violation of alveolar ventilation;
C. he increase in the mass of muscle fibers;
D. the reduction of glycogen in the heart muscle;
E. increase in blood pressure in the vessels.

21. In a patient there is a lack of cyanocobalamin and folic acid, which served as the basis
for violation of leukopoiesis. What changes may be in this case?
A. leukopenia.
B. basophilia;
C. hemophilia;
D. hyperemia;
E. eosinophilia.
21
22. In a patient S., are revealed such changes in peripheral blood: er. 3,2*1012 / l gems.
80 g / l, Lake. 25*109 / l. WBC: 5% basophils, eosinophils, 9%, 8% promyelocytes,
myelocytes 14%, metamyelocytes 22%, 17% stab, segmented 19%, lymphocytes 3%,
monocytes 3%. Determine the most probable pathology that fit this description.
A. chronic myelogenous leukemia;
B. erythromyelosis;
C. leukemoid reaction;
D. undifferentiated leucosis;
E. acute myeloid leukemia.

23. Patient P., revealed such changes in peripheral blood: er. 3,0*1012 / l gems. 80 g / l,
Lake. 21*109 / l. WBC: 0% basophils 0%, eosinophils, 0%, myeloblasts 54%,
promyelocytes 1% myelocytes 0% metamyelocytes 0%, .seg. 28%, stab 1%, lymphocytes
13%, monocytes 3%. Determine the most probable pathology, appropriate to the
description of the blood picture:
A. acute myeloid leukemia;
B. erythroleukemia;
C. leukemoid reaction;
D. undifferentiated leukemia;
E. chronic myelogenous leukemia.

24. In a patient C. are revealed the following changes in the peripheral blood: Er. 3,2
*1012 / L Leuk. 25*109 / L. Wbc: basophiles - 5%, eosinophils - 9% myeloblasts - 3%
promyelocytes - 8%; myelocytes - 11% metamyelocytes - 22% stab - 17% segmented -
19% lymphocytes - 3% monocytes - 3%. Determine the most probable pathology that
corresponds this description of the blood picture:
A. chronic myelogenous leukemia;
B. acute myeloid leukemia;
C. erythroleukemia;
D. leukemoid reaction;
E. undifferentiated leukemia.

25. In operation on liquidation of consequences of the accident at the nuclear power plant
workers received dose of 500 X-rays. Complains of headache, nausea, dizziness. What
changes in the number of leukocytes in a patient can be expected in 10 hours after
irradiation?
A. leukocytosis;
B. lymphocytosis;
C. leukopenia;
D. agranulocytosis;
E. leukemia.

26. In a patient with chronic leukemia sharply increased the body temperature, appeared
shortness of breath, severe muscle weakness at low loads, excessive sweating, coughing.
Pneumonia was revealed. Which of the mechanisms of leukemia impact on the body is
the basis for this complication in a patient?
A. immunodeficiency;
22
B. anemia;
C. internal bleeding;
D. tumor progression;
E. airway obstruction.

27. A 54-year-old man complains of general weakness, frequent colds, and bruises
constantly appearing on his body. Blood test: erythrocytes - 2.5 1012/L; Hb- 80 g/L;
color index - 0.9; reticulocytes - absent;
platelets - 50 109/L; leukocytes - 58 109/L; leukogram: · - 5%, eosinophils
· basocytes · -
15%, myeloblasts - 6%, myelocytes - 10%,
juvenile - 18%, stab neutrophils - 26%, segmented neutrophils - 10%, lymphocytes
- 8%, monocytes - 2%, ESR - 40 mm/hour. What hematologic conclusion can be made?
A. Chronic myelogenous leukemia
B. Leukemoid response
C. Myeloblastic leukemia
D. Chronic lymphocytic leukemia
E. Basophilic eosinophilic leukocytosis

28. 24 hours after an appendectomy the patient’s blood test shows neutrophilic
leukocytosis with a regenerative shift. What is the most likely mechanism of absolute
leukocytosis development in the patient’s peripheral blood?
A. Intensification of leukopoiesis
B. Leukocyte redistribution
C. Decreased leukocyte disintegration
D. Deceleration of leukocyte migration to the tissues
E. Immunity activation

29. A patient presents with lymphocytic-monocytic leukogram pattern. It is characteristic


of:
A. Chronic inflammatory process
B. Acute inflammatory process
C. Allergy
D. Chronic radiation sickness
E.-

30.A 59-year-oldwomanhas been hospializedinasurgicalwardduetoexacerbationof chronic


osteomyelitisof the left shin. Blood test: leukocytes - 15,0 · 10*9/l. Leukogram:
myelocytes - 0%, metamyelocytes 8%, stab neutrophils - 28%, segmented neutrophils -
32%, lymphocytes - 29%, monocytes - 3%. Such blood count would be called:
A. Regenerativeleft shift
B. Right shift
C. Hyperregenerative left shift
D. Degenerativeleft shift
E. Regenerative-degenerative left shift

31.Apatientis20yearsold,anathlete.He addressed a doctor with complaints of fatigue,


fever up to 38oC - 40 oC. Objectively: the liver and spleen are enlarged, lymph nodes on
palpation are slightly enlarged, dense, painless. Blood test: Нb- 100 g/l; erythrocytes - 2,9
23
· 1012/l; leukocytes 4,4 · 109/l. Leukogram: 68% of blast cells. Cytochemical
investigation of blast cells revealed negative reactions to glycogen, peroxidase, non-
specific esterase, lipids. Name this disease:
A. Acute undifferentiatedleukemia
B. Acute myeloid leukemia
C. Acute monoblastic leukemia
D. Acute lymphoblasticleukemia
E. Acute megakaryoblasticleukemia
32. A 49-year-old man with myocardial infarction has been admitted to a cardiology
department. What changes in the peripheral blood cells are induced by the necrotic
changes in the myocardium?
A. Neutrophilic leukocytosis
B. Monocytosis
C. Eosinophilia
D. Thrombocytopenia
E. Lymphopenia

33. The total number of leukocytes in the patient’s blood is 90 · 109/l. Leukogram:
eosinophils - 0%, basophils - 0%, juvenile - 0%, stab neutrophils - 2%, segmented
neutrophils - 20%, lymphoblasts - 1%, prolymphocytes 2%, lymphocytes - 70%,
monocytes 5%, Botkin-Gumprecht cells. Clinical examination revealed enlarged cervical
and submandibular lymph nodes. Such clinical presentations are typical of the following
pathology:
A. Chronic lympholeukosis
B. Acute lympholeukosis
C. Lymphogranulomatosis
D. Infectious mononucleosis
E. Chronic myeloleukosis

34. A patient, who had suffered severe blood loss three days ago, underwent blood test.
The following data was obtained in leukogram: leukocytes 12 · 109/l, basophils - 0,
eosinophils 3, myelocytes - 0, juvenile - 3, stab neutrophils - 12, segmented neutrophils -
62, lymphocytes - 16, monocytes 4. What change of leukocyte content occurred in this
case?
A. Neutrophilia with regenerative leftshift
B. Neutrophilia with degenerative leftshift
C. Neutrophilia with right-shift
D. Absolute lymphopenia
E. Absolute monocytopenia

Disorders of hemostasis

1. Punctata hemorrhage was found out in the patient after application of a tourniquet.
Disfunction of what blood cells was it connected with?
A. Platelets
B. Eosinophiles
C. Monocytes
D. Lymphocytes
24
E. Neutrophiles

2. A patient with tissue trauma was taken a blood sample for the determination of blood
clotting parameters. Specify the right sequence of extrinsic pathway activation.
A. III – VII – Xa
B. III – IV – Xa
C. IV – VIII: TF – Xa
D. IV – VII – Xa
E. III – VIII: TF – Xa

3. A 2-year-old child has got intestinal dysbacteriosis, which results in hemorrhagic


syndrome. What is the most likely cause of hemorrhage of the child?
A. Vitamin K insufficiency
B. Activation of tissue thromboplastin
C. PP hypovitaminosis
D. Fibrinogen deficiency
E. Hypocalcemia

4. Patients with bile ducts obstruction suffer from inhibition of blood coagulation,
bleedings as a result of low level of vitamin assimilation. What vitamin is in deficiency?
A. К
B. А
C. D
D. Е
E. Carotene

5. A patient was ill with burn disease that was complicated by DIC syndrome. What stage
of DIC syndrome can be suspected if it is known that the patient’s blood coagulates in
less than 3 minutes?
A. Hypercoagulation
B. Transition phase
C. Hypocoagulation
D. Fibrinolysis
E. Termina

6. A patient underwent a surgery for excision of a cyst on pancreas. After this he


developed hemorrhagic syndrome with apparent disorder of blood coagulation.
Development of this complication can be explained by:
A. Activation of fibrinolytic system
B. Insufficient fibrin production
C. Reduced number of thrombocytes
D. Activation of anticoagulation system
E. Activation of Christmas factor

7. A 70-year-old patient suffers from atherosclerosis complicated by the lower limb


thrombosis that has caused gangrene on his left toes. What is the most likely cause of the
thrombosis origin?
A. Thrombocyte adhesion
25
B. Prothrombinase activation
C. Transformation of prothrombin into thrombin
D. Transformation of fibrinogen into fibrin
E. Impaired heparin synthesis

8. A tooth extraction in a patient with chronic persistent hepatitis was complicated with
prolonged hemorrhage. What is the reason for the hemorrhagic syndrome?
A. Decrease in thrombin production
B. Increase in thromboplastin production
C. Decrease in fibrin production
D. Increase in fibrinogen synthesis
E. Fibrinolysis intensification

9. After implantation of a cardiac valve a young man constantly takes indirect


anticoagulants. His state was complicated by hemorrhage. What substance content has
decreased in blood?
A. Prothrombin
B. Haptoglobin
C. Heparin
D. Creatin
E. Ceruloplasmin

10. A disaster fighter at a nuclear power plant developed hemorrhagic syndrome on the
background of acute radiation disease. What is the most important factor of syndrome
pathogenesis?
A. Thrombocytopenia
B. Vascular wall damage
C. Increased activity of fibrinolysis factors
D. Increased activity of anticoagulative system factors
E. Decreased activity of coagulative factors

11. A 3-year-old boy with pronounced hemorrhagic syndrome doesn’t have


antihemophilic globulin A (factor VIII) in the blood plasma. Hemostasis has been
impaired at the following stage:
A. Internal mechanism of prothrombinase activation
B. External mechanism of prothrombinase activation
C. Conversion of prothrombin to thrombin
D. Conversion of fibrinogen to fibrin
E. Blood clot retraction

12. A 12-year-old patient has been admitted to a hospital for hemarthrosis of the knee
joint. From early childhood he suffers from frequent bleedings. Diagnose the boy’s
disease:
A. Hemophilia
B. Hemorrhagic vasculitis
C. Hemolytic anemia
D. B12 (folic acid)-deficiency anemia
E. Thrombocytopenic purpura
26
13. A patient is diagnosed with hereditary coagulopathy that is characterised by factor
VIII deficiency. Specify the phase of blood clotting during which coagulation will be
disrupted in the given case:
A. Thromboplastin formation
B. Thrombin formation
C. Fibrin formation
D. Clot retraction
E. –

14. A pharmaceutical factory worker complained of general weakness, epistaxis,


significant bleeding of gums, multiple subcutaneous hemorrhages. In the blood: E -
2,2*1012 / l, Hb -48g / l, LE - 2,1*109 / l neutropenia with a relative lymphocytosis,
platelets – 35*109 / l. What is the most probable pathogenesis of thrombocytopenia that
can be assumed in this patient?
A. decrease in platelet production;
B. increased platelet destruction;
C. The increased consumption of platelets;
D. reallocation of platelets;
E. the increased loss of platelets.

15. In a child with hemorrhagic syndrome was diagnosed with hemophilia B. By the
deficiency of which coagulation factor is it caused?
A. factor IX;
B. VIII factor;
C. VI factor;
D. factor XI;
E. XII factor.

16. In a boy of 9 years, who often has colds, have appeared small skin hemorrhages .
Violations of functions of internal organs are not found. What is the pathology of in
the boy?
A. thrombocytopenia;
B. aplastic anemia;
C. agranulocytosis;
D. hemophilia;
E. polycythemia vera.

17. In patient with chronic hepatitis was removed a tooth . Bleeding, which arose after
that, was not possible to stop for 2 hours. Conducted research of hemostasis established a
decrease in the content of several blood clotting factors. What type of hemostasis is
violated in this case?
A. coagulation;
B. platelet-vascular;
C. coagulation-platelet;
D. platelet;
E. vascular.

27
18. Male 52 years old complains of fatigue, headaches, bleedings of gums. Objectively:
the skin has a reddish-cyanotic color, the spleen is enlarged. Analysis of blood: Hb - 200
g / l, erythrocytes - 6.4 *1012 / L, color index - 0.96, leukocytes - 10.5 *109 / l, b. - 0%,
e. - 4%, p. - 7%, p. - 60%, lymph. - 25%, mon. - 4% the platelets – 500*109 / L, ESR - 1
mm / hr. After examination was diagnosed with erythremia. What pathological process is
the basis of this disease?
A. tumor growth.
B. hypoplasia;
C. hyperplasia;
D. hypertrophy;
E. inflammation;

19. In a patient are revealed multiple bruises on the body, the duration of bleeding by
Duque is 25 minutes, the number of blood platelets – 25*109 / l. What disease is
characterized by such symptoms?
A. hereditary defect of platelet formation;
B. hemophilia A;
C. von Willebrand disease;
D. avitaminosis C;
E. hemophilia B.

20. The patient of 30 years old has been suffering from migraine, due to it often takes
analgin. Recently, have appeared small skin hemorrhages, frequent epistaxis, platelet
count 30*109 / l, bleeding time is increased. What causes these disorders?
A. autoimmune thrombocytopenia;
B. emolytic anemia;
C. angiogemofiliya;
D. hemorrhagic vasculitis;
E. trombotsitopaty.

21. A patient is diagnosed with a hereditary form of coagulopathy, which manifests itself
by defective blood coagulation factor VIII. Please indicate in which phase of blood
coagulation primary hemostatic disorders occur in this case?
A. the formation of thromboplastin;
B. the formation of thrombin;
C. the formation of fibrin;
D. the clot retraction;
E. all wrong.

22. In a patient after surgical intervention on the pancreas developed a hemorrhagic


syndrome with severe violation of the third phase of blood coagulation. Specify the
possible mechanism of hemostatic disorders in this case:
A. increased fibrinolysis.
B. the weakening of fibrinolysis;
C. the excess of factor XII;
D. the excess of factor VIII;
E. The deficit of factor XII.

28
23. In a patient a bleeding time by Duque is 15 minutes. Coagulation time 6 min.
Violation of which link of hemostasis is possible to assume in the patient?
A. vascular platelet;
B. coagulative, the I phase;
C. coagulative, phase II;
D. coagulative, phase III;
E. fibrinolysis.

24. After implantation of artificial heart valve to a patient he had a tendency to bleeding.
The platelet count in the blood is reduced. Thrombocytopenia in this patient can be
explained by
A. mechanical destruction of platelets;
B. the adhesion of platelets on the valve;
C. toxic lesion of platelets;
D. defeat of platelets by autoantibodies;
E. deficit of thrombocytopoietic factor.

25. In a patient of 45 years old was found a tumor of the pancreatic head, which is
accompanied by steatorrhea and hemorrhages on the skin. In the blood - reduction of
prothrombin and other coagulation factors. What is the mechanism of occurrence of
hemorrhages?
A. avitaminosis K;
B. improving of urobilin formation;
C. increase in the bile secretion ;
D. increase in the secretion of proteolytic enzymes;
E. reduction in secretin production.

26. In a patient during the examination was revealed a thrombocytopenia, which emerged
afterwards increased damage of platelets. To what group of thrombocytopenia does it
belong?
A. immune;
B. deficient;
C. disregulatory;
D. myelotoxic;
E. utilization.

27. In a patient of 43 years old, on the background of septic shock there is a


thrombocytopenia, a decrease of fibrinogen, the appearance of fibrin degradation
products in the blood, the appearance of petechial hemorrhages. Specify reason of
occurrence of these changes.
A. DIC;
B. hemorrhagic diathesis;
C. autoimmune thrombocytopenia;
D. violation of platelet production;
E. exogenous intoxication.

28. In a study of patients with hemophilia revealed a change in some parameters of blood.
Which of the following symptoms corresponds to this disease?
29
A. increased blood clotting time;
B. polycythemia;
C. eosinophilia;
D. afibrinogenemia;
E. thrombocytopenia.

29. A 28-year-old patient complains of frequent gingival hemorrhages. Blood test


revealed the clotting factor II (prothrombin) deficiency. What phase of blood coagulation
is impaired in this patient?
A. Thrombin generation
B. Vascular-platelet haemostasis
C. Clot retraction
D. Fibrinolysis
E. -

30. A 60-year-old woman with hepatocirrhosis developed hemorrhagic syndrome. What


mechanism leads to the development of this condition?
A. Decreased synthesis of prothrombin and fibrinogen
B. Increased portal venous pressure
C. Deceased blood oncotic pressure
D. Reduction of hepatic glycogen stores
E. Emergence of neurotoxins in the blood

31. After pancreatic surgery the patient developed hemorrhagic syndrome with disturbed
3rd stage of blood clotting. What will be the most likely mechanism of the hemostatic
disorder?
A. Fibrinolysis activation
B. Decrease of prothrombinsynthesis
C. Decrease of fibrinogen synthesis
D. Qualitative abnormalities of fibrinogenesis
E. Fibrin-stabilizingfactor deficiency

32. A patient visited a dentist to extract a tooth. After the tooth had been extracted,
bleeding from the tooth socket continued for 15 minutes. Anamnesis states that the
patientsuffersfromactivechronichepatitis. What phenomenon can extend the time of
hemorrhage?
A. Decreaseof fibrinogen content in blood
B. Thrombocytopenia
C. Hypocalcemia
D. Increased activity of anticoagulation system
E. Decrease of albuminecontent in blood

33.Apatienthaspetechialhemorrhages onthegums,hardandsoftpalate,buccal mucosa. This


is caused by the dysfunction of the following blood corpuscles:
A. Platelets
B. Eosinophils
C. Monocytes
D. Lymphocytes
30
E. Erythrocytes

34. Which of the named below is the substrate of activated Christmas factor that takes
part in blood coagulation?
A. Factor X
B. Vitamin K
C. Fibrinogen
D. Fibrin
E. Thrombin

Blood circulation insufficiency

1. The process of heart transplantation determined the viability of myocardial cells. The
determination of what myocardium parameter is the most important?
A. Rest potential of cardiomyocytes
B. Heart temperature
C. Concentration of oxygen in heart vessels
D. Concentration of calcium-ions in myofibrils
E. Concentration of Ca-ions in heart vessels

2. Dystrophic alterations of heart are accompanied with dilation of heart cavities,


decreased force of heart contractions, increased blood volume that remains during systole
in the heart cavity, vein overfill. What heart condition is it typical for?
A. Myogenic dilatation
B. Tonogenic dilatation
C. Emergency stage of hyperfunction and hypertrophy
D. Cardiosclerosis
E. Cardiac tamponade

3. A patient who suffers from heart failure has enlarged liver, edemata of lower
extremities, ascites. What is the leading mechanism in the development of this edema?
A. Hydrodynamic
B. Colloid osmotic
C. Lymphogenous
D. Membranogenic
E. –

4. A patient who suffers from acute myocarditis has clinical signs of cardiogenic shock.
What of the under-mentioned pathogenetic mechanisms plays the main part in shock
development?
A. Disturbance of pumping ability of heart
B. Depositing of blood in organs
C. Reduction of diastolic flow to the heart
D. Decrease of vascular tone
E. Increase of peripheral vascular resistance

31
5. In course of a preventive examination of a miner a doctor revealed changes of
cardiovascular fitness which was indicative of cardiac insufficiency at the compensation
stage. What is the main proof of cardiac compensation?
A. Myocardium hypertrophy
B. Tachycardia
C. Rise of arterial pressure
D. Dyspnea
E. Cyanosis

6. A patient ill with essential arterial hypertension had a hypertensive crisis that resulted
in an attack of cardiac asthma. What is the leading mechanism of cardiac insufficiency in
this case?
A. Heart overload caused by high pressure
B. Heart overload caused by increased blood volume
C. Absolute coronary insufficiency
D. Myocardium damage
E. Blood supply disturbance

7. ECG of a 44-year-old patient shows signs of hypertrophy of both ventricles and the
right atrium. The patient was diagnosed with the tricuspid valve insufficiency. What
pathogenetic variant of cardiac dysfunction is usually observed in case of such
insufficiency?
A. Heart overload by volume
B. Heart overload by resistance
C. Primary myocardial insufficiency
D. Coronary insufficiency
E. Cardiac tamponade

8. The minute blood volume in a patient with transplanted heart has increased as a result
of physical activity. What regulative mechanism is responsible for these changes?
A. Catecholamines
B. Sympathetic unconditioned reflexes
C. Parasympathetic unconditioned reflexes
D. Sympathetic conditioned reflexes
E. Parasympathetic conditioned reflexes

9. A 50-year-old patient complains about general weakness, appetite loss and cardiac
arrhythmia. The patient presents with muscle hypotonia, flaccid paralyses, weakened
peristaltic activity of the bowels. Such condition might be caused by:
A. Hypokaliemia
B. Hypoproteinemia
C. Hyperkaliemia
D. Hypophosphatemia
E. Hyponatremia

10. A 56 year old patient suffering from cardiac insufficiency has edema of feet and
shins, edematous skin is pale and cold. What is the leding mechanism of edema
pathogenesis?
32
A. Rise of hydrostatic pressure in venules
B. Drop of oncotic pessure in capillaries
C. Increase of capillary permeability
D. Disorder of lymph outflow
E. Positive water balance

11. A 63 year old male patient who had been suffering from chronic diffuse obstructive
disease, pulmonary emphysema, for 15 years died from cardiac insufficiency. Autopsy
revealed nutmeg liver cirrhosis, cyanotic induration of kidneys and spleen, ascites,
edemata of lower limbs. These changes of internal organs are typical for the following
disease:
A. Chronic right-ventricular insufficiency
B. Acute right-ventricular insufficiency
C. Chronic left-ventricular insufficiency
D. Acute left-ventricular insufficiency
E. General cardiac insufficiency

12. The patient with acute miocardial infarction was given intravenously different
solutions during 8 hours with medical dropper 1500 ml and oxygen intranasally. He died
because of pulmonary edema. What caused the pulmonary edema?
A. Volume overload of the left ventricular
B. Decreased oncotic pressure due to hemodilution
C. Allergic reaction
D. Neurogenic reaction
E. Inhalation of the oxygen

13. During the fight, a man had a cardiac arrest due to the strong blow to the upper region
of the anterior abdominal wall. Which of the following mechanisms has led to the cardiac
arrest?
A. Parasympathetic unconditioned reflexes
B. Sympathetic unconditioned reflexes
C. Parasympathetic conditioned reflexes
D. Sympathetic conditioned reflexes
E. Peripheral reflexes

14. In a woman who has suffered of severe hypertension for 15 years, recently appeared
shortness of breath, palpitations, slightly decreased systolic blood pressure. What is the
main mechanism of occurrence of heart failure in a patient?
A. overload of the heart by increased resistance to blood flow;
B. overload of heart by increased blood volume;
C. myocardial damage;
D. violation of the impulse conduction along the myocardium;
E. dysregulation of cardiac activity.

15. In a patient with diphtheria developed a myocarditis with manifestations of blood


circulation failure. What hemodynamic violation is typical for this patient?
A. decrease in maximum arterial pressure;
B. the decrease in minimum arterial pressure;
33
C. the increase in the maximum blood pressure.
D. the increase in blood flow circulation;
E. the decrease in venous pressure.

16. In a patient C. with rheumatism, 7 years ago was diagnosed with heart defect. For
what type of pathology that is observed in this patient, long-term compensation is
provided by intracardiac heterometrycal mechanism of adaptation to the increased load
on the heart?
A. mitral valve insufficiency;
B. an arterial hypertensia;
C. aortic stenosis;
D. mitral stenosis;
E. stenosis of the pulmonary artery.

17. The patient complains of shortness of breath, cough with bloody sputum, a feeling of
fatigue at fast walking. In the anamnesis - rheumatism. At what form of heart disease in a
patient there is a intracardiac homeometrical mechanism of adaptation to increased load?
A. mitral stenosis;
B. mitral insufficiency;
C. hypervolemia;
D. failure of valves of the pulmonary artery;
E. stenosis of the aortic mouth.

18. In a patient of 50 years old with postinfarction cardiosclerosis on the basis of


coronary heart disease there are manifestations of chronic heart failure. Specify the stage
of compensatory hyperfunction of the heart in this case:
A. phase stage of exhaustion;
B. stage of resistant hyperfunction;
C. stage of completion of hypertrophy;
D. Stage of myogenic dilatation;
E. emergency.

19. A man of 25 years old complains of persistent pain in the heart area, shortness of
breath while moving, general weakness. Objectively: skin is pale, cold, acrocyanosis.
Pulse 96 beats. / Min., BP - 105/70 mm Hg. The border of the heart is shifted to the left
by 2 cm. I tone over the apex of the heart is weakened, systolic murmur over the apex.
Diagnosed with mitral valve insufficiency. What caused the poor circulation?
A. overload of myocardium by increased blood volume;
B. myocardial damage;
C. the reduction of circulating blood volume;
D. overload of myocardium by increased resistance;
E. the increase in the volume of the vascular channel.

20. A woman of 20 years old complains of general weakness, persistent pain in the heart
area, shortness of breath during physical exertion. Objectively: skin is pale and cold,
acrocyanosis. Pulse 86 in 1 min., Blood pressure - 110/80 mm Hg. The border of the
heart is shifted to the left by 1 cm. I tone over the apex of the heart is weakened, systolic

34
murmur over the aorta. Roentgenologically - aortic configuration of the heart. Diagnosed
with aortic valve stenosis. What caused the poor blood circulation?
A. overload of myocardium by increased resistance to blood flow;
B. myocardial damage;
C. overload of myocardium by increased blood volume;
D. the reduction in circulating blood volume;
E. the myocardial ischemia.

21. The patient is marked by cyanosis, enlarged liver, swelling of the lower extremities as
a result of right heart failure. What is the cause of right ventricular failure?
A. hypertension of the pulmonary circulation;
B. functional pulmonary shunting;
C. increase in venous pressure;
D. cardiogenic cirrhosis of the liver;
E. hypercatecholaemia.

22. The patient of 59 years old is hospitalized at cardiological department in serious


condition with a diagnosis of acute myocardial infarction in the posterior wall of the left
ventricle and septum, starting pulmonary edema. What is the primary mechanism that
causes the development of pulmonary edema in a patient?
A. left ventricular failure;
B. pulmonary arterial hypertension;
C. pulmonary venous hypertension;
D. hypoxemia;
E. reduction of the alveolar-capillary diffusion of oxygen.

23. In a patient K., 72 years old with chronic heart failure was revealed the development
of edema. Explain the mechanism of it's development.
A. blood stasis;
B. violation of proteinformation function of a liver;
C. hypersecretion of vasopressin;
D. proteinuria;
E. hyperaldosteronism.

24. In a boy of 5 years old with congenital stenosis of the pulmonary valve while walking
increased dyspnea, appeared cyanosis of the face and he lost consciousness. What is the
main mechanism of the development of this condition?
A. acute hypoxia of the brain;
B. decrease in blood pressure;
C. violation of pulmonary ventilation;
D. violation of the diffusion of gases in the lungs;
E. expansion of peripheral vessels.

25. In a patient of 56 years old, suffering from heart failure, there is a swelling of feet and
shins, skin is pale and cold in the edema area. What is the main link in the pathogenesis
of edema in a patient?
A. increase in hydrostatic pressure in the capillaries;
B. decrease in oncotic pressure in the capillaries;
35
C. increase of capillary permeability;
D. violation of lymphatic drainage;
E. positive water balance.

26. In a patient of 25 years old after acute endocarditis appeared aortic valves
insufficiency, which is caused by hypertrophy of the left ventricle of the heart. After eight
weeks, the mass of the heart was increased by 40% and its growth stopped. What stopped
the growth of myocardial mass?
A. reduction of load per unit of weight of muscle;
B. worsening of energy supply of myocardiocytes;
C. violation of regulatory supply of the heart;
D. reduction in protein synthesis and deterioration of the plastic supply of
E. myocardiocytes;
F. violation of vascular supply of the heart.

27. In a woman of 55 years old after a prolonged uterine bleeding appeared an iron
deficiency anemia. At the same time it has been revealed a dystrophic phenomena in the
myocardium, which had not previously been observed. What could cause a myocardial
dystrophy?
A. reduction of activity of ferriferous enzymes in tissues;
B. disappearance of hemosiderin in macrophages of the liver;
C. increase in the concentration of transferrin in the blood plasma;
D. reduction in the number of siderocytes and sideroblasts in the bone marrow;
E. increase in the intensity of glycolysis in erythrocytes.

28. In a patient with hypertension is revealed a significant increase of left ventricular


myocardial mass. This happened due to:
A. increase in the volume of cardiomyocytes;
B. proliferation of connective tissue;
C. water retention in the myocardium;
D. fatty infiltration of myocardium;
E. increase in the number of cardiomyocytes.

29. The patient of 45 years old complains of shortness of breath at low physical exertion,
swelling of the legs, in the anamnesis: frequent sore throats, sick for two years.
Diagnosed with insufficiency of blood circulation. What hemodynamic decompensation
of heart is observed in this case?
A. decrease in cardiac output;
B. reduction of venous pressure;
C. reduction of blood volume;
D. increase of arterial pressure;
E. tachycardia.

30. A man of 25 years old revealed mitral valve insufficiency without circulatory
disorders. What urgent mechanism provides cardiac compensation?
A. heterometrical;
B. homeometrical;
C. myogenic dilatation;
36
D. reduction of heart weight;
E. increased synthesis of catecholamines.

31. In a girl of 15 years old is diagnosed an aortic stenosis, but circulatory disorder was
not revealed. What urgent mechanism provides cardiac compensation?
A. homeometrical;
B. increased blood pressure;
C. heterometrical;
D. reduction of heart weight;
E. myogenic dilatation.

32. In a patient with alcoholic cirrhosis there are complaints of general weakness,
shortness of breath. A reduction in blood pressure, ascites, expansion of superficial veins
of the front wall of the abdomen, splenomegaly. What hemodynamic instability is
observed in a patient?
A. portal hypertension syndrome;
B. failure of the left ventricle of the heart;
C. failure of the right ventricle of the heart;
D. collapse;
E. total heart failure.

33. Patient M. suffers from chronic heart failure. Which of the following hemodynamic
parameters are leading signs of failure?
A. reduction of stroke volume;
B. tonogenic dilatation;
C. increase of peripheral vascular resistance;
D. development of tachycardia;
E. increase of central venous pressure.

34. In a patient with essential hypertension at ultrasonic research is revealed a dilatation


of the heart cavities. Which of the following signs indicates the development of
tonogenic dilation?
A. expansion of heart cavities with increase of stroke volume ;
B. expansion of the cavity with a decrease in stroke volume;
C. expansion of the cavity without changing the stroke volume;
D. evenly expansion of the boundaries of the heart;
E. uneven expansion of the boundaries of the heart.

35. A patient, 44 years old, complains of shortness of breath, palpitations, pain in the
right intercostal space, swelling of the legs. On ECG - signs of hypertrophy of both
ventricles and the right atrium. Diagnosed with tricuspid regurgitation. What pathogenic
variant of this failure?
A. volume overload of the heart;
B. overload of heart by pressure;
C. primary myocardial failure;
D. coronary insufficiency;
E. cardiac tamponade.

37
36. In1 hour after banding, which constricts the aorta in a dog sharply increased power
and rate of heart contractions, blood volume and left ventricular wall thickness were not
different from baseline values. What stage of myocardial hypertrophy is observed in an
animal?
A. emergency;
B. decompensation;
C. progressive cardiosclerosis;
D. relatively resistant hyperfunction;
E. completion of hypertrophy.

37. At reproduction of hypertension in a dog in 1 month the left ventricular wall


thickness increased by 1.7 times and the blood volume is not changed in comparison with
the original data. What stage of myocardial hypertrophy is observed in an animal?
A. completed hypertrophy;
B. initial;
C. progressive cardiosclerosis;
D. emergency;
E. decompensation.

38. In a patient with mitral valve insufficiency occurred a hypertrophy of the left
ventricle. What is the trigger mechanism in the development of hypertrophy?
A. activation of the genetic apparatus;
B. increase in consumption of fatty acids;
C. increase in the intensity of cellular respiration;
D. activation of glycolysis;
E. increase in income of Ca2 + into the cell.

39. In a patient with essential hypertension developed a myocardial hypertrophy. What is


the main mechanism of increasing the volume of myocardiocytes?
A. increased synthesis of contractile proteins;
B. increase in myocardial capillary network;
C. increase of the endoplasmic reticulum;
D. increase in the number of mitochondria;
E. myocardiocytes hyperplasia.

40. In a patient after a moderate physical activity there was an urgent dyspnoea. Cough
with bloody sputum. On examination - cyanosis of the lips, at auscultation - variegated
rhonchi throughout the lungs. What form of heart failure is possible to suspect in this
patient?
A. total heart failure;
B. chronic left ventricular failure;
C. acute left ventricular failure;
D. acute right ventricular failure;
E. chronic right ventricular failure.

41. Degenerative changes in the heart muscle are accompanied by expansion of cavities
of the heart, decreased strength of heart contractions, increase in blood volume that
remains during systole in the cavities of the heart, veins overflow. All this is typical for:
38
A. myogenic dilatation;
B. tonogenic dilatation;
C. emergency phase of myocardial hypertrophy;
D. stage of sequential exhaustion and progressive cardiosclerosis of hypertrophied
myocardium;
E. cardiac tamponade.

42. Patients with acute myocardial infarction intravenously was injected 1500 ml of
different solutions within 8 hours, oxygen intranasally. Death was caused by a pulmonary
edema. What caused a pulmonary edema?
A. left ventricular overload of the heart by volume;
B. decrease in oncotic pressure due to hemodilution;
C. allergic reaction;
D. neurogenic response;
E. oxygen inhalation.

43. In the course of experiment the vagus nerve of the test animal was severed, which
resulted in the animal developing constant tachycardia. What effect of parasympathetic
nervous system on cardiac performance is demonstrated by this experiment?
A. Inhibition
B. Stimulation
C. Stimulus summation
D. Paradoxical response
E. Mixed effect

44. Due to blood loss the circulating blood volume of a patient decreased. How will it
affect the blood pressure in this patient?
A. Systolic and diastolic pressure will decrease
B. Only systolic pressure will decrease
C. Only diastolicpressure will decrease
D. Systolic pressure will decrease, while diastolicwillincrease
E. Diastolic pressure will decrease, while systolic will increase

45. A patient has a history of chronic heart failure. Which of the following hemodynamic
parameters is a major symptom of cardiac decompensation development?
A. Decreased stroke volume
B. Tachycardia development
C. Tonogenic dilatation
D. Increased peripheral vascular resistance
E. Increased central venous pressure

46. In the course of an experiment researchers stimulate a branch of the sympathetic


nerve that innervates heart. What alterations of cardiac activity should be registered? A.
Increase in heart rate and heart force
B. Decrease in heart force
C. Increase in heart rate
D. Increase in heart force
E. Increase in arterial pressure
39
47. An athlete (a long-distance runner) during competition has developed acute heart
failure. This pathology developed due to:
A.Volume overload
B. Coronary blood flow disturbance
C. Direct damage to myocardium
D. Pericardial pathology
E. Pressure overload

48. An athlete (long-distance runner)during a contest developed a case of acute cardiac


insufficiency. This pathology resulted from:
A. Cardiac volumeoverload
B. Disrupted coronary circulation
C. Direct damageto myocardium
D. Pericardium pathology
E. Cardiac pressure overload

49. Patient’s systolic blood pressure is 90 mm Hg, diastolic - 70 mm Hg. Such blood
pressure is caused by decrease of the followingfactor:
A. Pumping abilityof the left heart
B. Pumping abilityof the right heart
C. Aortic compliance
D. Total peripheral resistance
E. Vascular tone

50. A patient has a history of chronic heart failure. Which of the following hemodynamic
parameters is a major symptom of cardiac decompensation development?
A. Decreased stroke volume
B. Tachycardia development
C. Tonogenic dilatation
D. Increased peripheral vascular resistance
E. Increased central venous pressure

51. After a serious psychoemotional stress a 48-year-old patient suddenly developed


acute heart ache irradiating to the left arm. Nitroglycerine relieved the pain attack after 10
minutes. What is the leading pathogenetic mechanism of this process development? A.
Spasm of coronary arteries
B. Dilatation of peripheral vessels
C. Obstruction of coronary vessels
D. Compression of coronary vessels
E. Increase in myocardial oxygen consumption

Сoronary insufficiency. Cardiac arrhythmias

1. Electrocardiogram of a 45-year-old man showed absence of P-wave in all the leads.


What part of the conducting system is blocked?
1. Sinu-atrial node
2. Atrioventricular node
40
3. Common branch of the bundle of His
4. Branches of the bundle of His
5. Purkinje’s fibres

2. A person has steady HR not exceeding 40 bpm. What is the pacemaker of the heart
rhythm in this person?
A. Atrioventricular node
B. Sinoatrial node
C. His’ bundle
D. Branches of His’ bundle
E. Purkinye’ fibers

3. The calcium canals of cardiomyocytes have been blocked on an isolated rabbit’s heart.
What changes in the heart’s activity can result from it?
A. Decreased rate and force of heart beat
B. Decreased heart beat rate
C. Decreased force of the contraction
D. Heart stops in systole
E. Heart stops in diastole

4. A peripheral segment of vagus nerve on a dog’s neck was being stimulated in course of
an experiment. The following changes of cardiac activity could be meanwhile observed:
A. Heart rate fall
B. Heart hurry
C. Enhancement of atrioventricular conduction
D. Heart rate and heart force amplification
E. Increased excitability of myocardium

5. Heart rate of a man permanently equals 40 beats pro minute. What is the pacemaker?
A. Atriventricular node
B. Sinoatrial node
C. His’ bundle
D. His’ bundle branches
E. Purkinje’s fibers

6. A 49 y.o. woman consulted a doctor about heightened fatigue and dyspnea during
physical activity. ECG: heart rate is 50/min, PQ is extended, QRS is unchanged, P wave
quanity exceeds quantity of QRS complexes. What type of arrhythmia does the patient
have?
A. Atrioventricular block
B. Extrasystole
C. Sinus bradycardia
D. Ciliary arhythmia
E. Sinoatrial block

7. A patient has extrasystole. ECG shows no P wave, QRS complex is deformed, there is
a full compensatory pause. What extrasystoles are these?
A. Ventricular
41
B. Atrial
C. Atrioventricular
D. Sinus
E. –

8. ECG of a patient with hyperfunction of thyroid gland showed heart hurry. It is


indicated by depression of the following ECG element:
A. R − R interval
B. P − Q segment
C. P − Q interval
D. P − T interval
E. QRS complex

9. Vagus branches that innervate heart are being stimulated in course of an experiment.
As a result of it the excitement conduction from atria to the ventricles was brought to a
stop. It is caused by electrophysical changes in the following structures:
A. Atrioventricular node
B. His’ bundle
C. Sinoatrial node
D. Ventricles
E. Atria

10. A cardiac electric stimulator was implanted to a 75 year old man with heart rate of 40
bpm. Thereafter the heart rate rose up to 70 bpm. The electric stimulator has undertaken
the function of the following heart part:
A. Sinoatrial node
B. Atrioventricular node
C. His’ bundle branches
D. His’ bundle fibers
E. Purkinje’s fibers

11. A 49-year-old patient consulted a doctor about increased fatigability and dyspnea
provoked by physical activity. ECG results: heart rate - 50/min, PQ interval is prolonged,
QRS- complex is unchanged, the number of P-waves exceeds the number of QRS-
complexes. What type of arrhythmia is it?
A. Atrioventricular block
B. Extrasystole
C. Sinus bradycardia
D. Ciliary arrhythmia
E. Sinoatrial block

12. A 45 year old patient was admitted to the cardiological department. ECG data:
negative P wave overlaps QRS complex, diastolic interval is prolonged after extrasystole.
What type of extrasystole is it?
A. Atrioventricular
B. Sinus
C. Atrial
D. Ventricular
42
E. Bundle-branch

13. Analysis of the ECG revealed the missing of several PQRST cycles. The remaining
waves and complexes are not changed. Specify the type of arrhythmia:
A. Sinoatrial block
B. Atrial fibrillation
C. Atrioventricular block
D. Atrial premature beat
E. Intra-atrial block

14. ECG of a patient displays an abnormally long R wave (up to 0,18 s). This is caused
by a decrease in the conduction velocity of the following heart structures:
A. Ventricles
B. Atria
C. Atrio-ventricular node
D. Right ventricle
E. Left ventricle

15. Since a patient has had myocardial infarction, atria and ventricles contract
independently from each other with a frequency of 60-70 and 35-40 per minute. Specify
the type of heart block in this case:
A. Complete atrioventricular
B. Partial atrioventricular
C. Sino-atrial
D. Intra-atrial
E. Intraventricular

16. A patient who had been continuously taking drugs blocking the production of
angiotensin II developed bradycardia and arrhythmia. A likely cause of these disorders is:
A. Hyperkalemia
B. Hypokalemia
C. Hypernatremia
D. Hypocalcemia
E. Hypercalcemia

17. A patient complains of palpitation after stress. The pulse is 104 bpm, P-Q=0,12
seconds, there are no changes of QRS complex. What type of arrhythmia does the patient
have?
A. Sinus tachycardia
B. Sinus bradycardia
C. Sinus arrhythmia
D. Ciliary arrhythmia
E. Extrasystole

18. Examination of an isolated cardiomyocyte revealed that it didn’t generate excitation


impulses automatically. This cardiomyocyte was obtained from:
A. Ventricles
B. Sinoatrial node
43
C. Atrioventricular node
D. His’ bundle
E. Purkinje’s fibers

19. ECG of a patient shows such alterations: P-wave is normal, P − Q-interval is short,
ventricular QRST complex is wide, R-wave is double-peak or two-phase. What form of
arrhythmia is it?
A. WPW syndrome (Wolff-ParkinsonWhite)
B. Frederick’s syndrome (atrial flutter)
C. Atrioventricular block
D. Ventricular fibrillation
E. Ciliary arrhythmia

20. In a healthy adult speed of the excitement conduction through the atrioventricular
node is 0,02-0,05 m/sec. Atrioventricular delay enables:
A. Sequence of atrial and ventricular contractions
B. Simultaneity of both atria contractions
C. Simultaneity of both ventricles contractions
D. Sufficient force of atrial contractions
E. Sufficient force of ventricular contractions

21. Marked increase of activity of МВ- forms of CPK (creatinephosphokinase) and LDH-
1 was revealed by examination of the patient’s blood. What is the most probable
pathology?
A. Miocardial infarction
B. Hepatitis
C. Rheumatism
D. Pancreatitis
E. Cholecystitis

22. A 70 year old man is ill with vascular atherosclerosis of lower extremities and
coronary heart disease. Examination revealed disturbance of lipidic blood composition.
The main factor of atherosclerosis pathogenesis is the excess of the following
lipoproteins:
A. Low-density lipoproteins
B. Cholesterol
C. High-density lipoproteins
D. Intermediate density lipoproteins
E. Chylomicrons

23. After a serious psychoemotional stress a 48 year old patient suddenly developed acute
heart ache irradiating to the left arm. Nitroglycerine relieved pain after 10 minutes. What
is the leading pathogenetic mechanism of this process development?
A. Spasm of coronary arteries
B. Dilatation of peripheral vessels
C. Obstruction of coronary vessels
D. Compression of coronary vessels
E. Increase in myocardial oxygen consumption
44
24. A patient with extensive myocardial infarction has developed heart failure. What
pathogenetic mechanism contributed to the development of heart failure in the patient?
A. Reduction in the mass of functioning myocardiocytes
B. Pressure overload
C. Volume overload
D. Acute cardiac tamponade
E. Myocardial reperfusion injury

25. In a patient, who six weeks ago had a myocardial infarction, is diagnosed with
Dresler syndrome with characteristic triad of pericarditis, pleurisy, pneumonia. What is
the main mechanism of this complication?
A. sensitization of the organism by antigens of infarction;
B. reduction of resistance to infectious agents;
C. activation of saprophytic microflora;
D. intoxication of organism by products of necrosis;
E. release of myocardial enzymes to the blood.

26. In a patients with coronary heart disease suddenly appeared a severe attack of angina.
The face is pale, the skin is wet, cold, blood pressure 70/50 mm Hg. Art. arrythmia.
Diagnosed with myocardial infarction and cardiogenic shock. What is the primary circuit
of pathogenesis:
A. decrease in cardiac output;
B. extrasystoles;
C. toxemia;
D. pain syndrome;
E. hypotension.

27. Coronary artery thrombosis caused a myocardial infarction. What mechanisms of cell
damage are dominant in this disease?
A. calcium;
B. lipid;
C. acidotic;
D. electrolytic-osmotic;
E. protein.

28. In 3 weeks after an acute myocardial infarction in a patient appeared pains in heart
and joints, lung inflammation. What is the main mechanism in the development of
postinfarction syndrome of Dressler in this patient?
A. autoimmune inflammation;
B. secondary infection;
C. myocardial ischemia;
D. vascular thrombosis;
E. resorption of proteins from necrotic myocardium.

29. A 17-year-old girl suffers from periodical palpitations that last several minutes. Her
heart rate is 200/min., rhythmic. What heart rhythm disorder developed in this patient?
A. Paroxysmal tachycardia
45
B. Sinus tachycardia
C. Sinus bradycardia
D. Extrasystole
E. Atrioventricular block

30. ECG of the patient shows increased duration of the QRS complex. What is the most
likely cause?
A. Increased period of ventricular excitation
B. Disturbed conduction in the atrioventricular node
C. Increased atrial excitability
D. Increased atrial and ventricular excitability
E. Increased period of atrial excitation

31. A 48-year-old man is unconscious. He has a history of several syncopal episodes with
convulsions. ECG shows deformed QRS complexes unconnected with P waves, atrial
contractions are approximately 70/min., ventricular contractions - 25-30/min. Name the
type of arrhythmia in this case:
A. Complete atrioventricular block
B. First-degree atrioventricular block
C. Second-degree atrioventricular block
D. Intraatrial block
E. Intraventricular block

32. A 67-year-old man was delivered to a cardiology department with complaints of


periodical pains in his heart, dyspnea caused by even slight exertion, cyanosis and
edemas. ECG shows additional excitations of heart ventricles. Name this type of rhythm
disturbance:
A. Extrasystole
B. Bradycardia
C. Tachycardia
D. Flutter
E. Fibrillation

33. A 15-year-old teenager complains of lack of air, general weakness, palpitations. Heart
rate is 130/min., BP is 100/60 mm Hg. ECG: QRS complex has normal
shapeandduration.ThenumberofPwaves and ventricular complexes is equal, T wave
merges with P wave. What type of cardiac arrhythmiais observed in the teenager?
A. Sinus tachycardia
B. Sinus extrasystole
C. Atrial fibrillation
D. Atrial thrill
E. Paroxysmal atrial tachycardia

34.A 67-year-oldmanwasdeliveredto the cardiology unit with complaints of periodical


pain in the heart, dyspnea after even insignificantphysicalexertion,cyanosis,and
edemas.ECGrevealedadditionalcontractions of the heart ventricles. Name this type of
rhythm disturbance:
A. Extrasystole
46
B. Bradycardia
C. Tachycardia
D. Flutter
E. Fibrillation

35. A patient complains of palpitations after stress. Pulse is 104/min., P-Q=0,12 seconds,
there are no changes in QRS complex. What type of arrhythmia does the patient have?
A. Sinus tachycardia
B. Sinus bradycardia
C. Sinus arrhythmia
D. Ciliaryarrhythmia
E. Extrasystole

36. A 67-year-old patient complains of periodic heartache, dyspnea during light physical
activities. ECG reveals extraordinary contractions of heart ventricles. Such arrhythmia is
called:
A. Extrasystole
B. Bradycardia
C. Tachycardia
D. Flutter
E. Fibrillation

Pathophysiology of blood vessels

1. Arterial pressure of a surgeon who performed a long operation rised up to 140/110 mm


Hg. What changes of humoral regulation could have caused the rise of arterial pressure in
this case?
A. Activation of sympathoadrenal system
B. Activation of formation and excretion of aldosterone
C. Activation of renin angiotensive system
D. Activation of kallikrein kinin system
E. Inhibition of sympathoadrenal system

2. An aged man had raise of arterial pressure under a stress. It was caused by activation
of:
A. Sympathoadrenal system
B. Parasympathetic nucleus of vagus
C. Functions of thyroid gland
D. Functions of adrenal cortex
E. Hypophysis function

3. Prophylactic medical examination of a 36 year old driver revealed that his AP was
150/90 mm Hg. At the end of working day he usually hears ear noise, feels slight
indisposition that passes after some rest. He was diagnosed with essential hypertension.
What is the leading pathogenetic mechanism in this case?
A. Neurogenetic
B. Nephric
C. Humoral
47
D. Endocrinal
E. Reflexogenic

4. A patient has osmotic pressure of blood plasma at the rate of 350 mOsmol/l (norm is
300 mOsmol/l). This will cause hypersecretion of the following hormone:
A. Vasopressin
B. Aldosterone
C. Cortisol
D. Adrenocorticotropin
E. Natriuretic

5.In response to a change in body position from horizontal to vertical blood circulation
system develops reflectory pressor reaction. Which of the following is its compulsory
component?
A. Systemic constriction of the venous vessels
B. Systemic dilatation of the arterial resistive vessels C
C. Decrease in the circulating blood volume
D. Increase in the heart rate
E. Weakening of the pumbing ability of heart

6. An adult man presents with systemic arterial pressure drop from 120/70 to 90/50 mm
Hg. This resulted in reflex vasoconstriction. Vasoconstriction will be minimal in the
following organ:
A. Heart
B. Skin
C. Bowels
D. Skeletal muscles
E. Liver

7. A month after surgical constriction of rabbit’s renal artery the considerable increase of
systematic arterial pressure was observed. What of the following regulation mechanisms
caused the animal’s pressure change?
A. Angiotensin-II
B. Vasopressin
C. Adrenaline
D. Noradrenaline
E. Serotonin

8. A patient with constant headaches, pain in the occipital region, tinnitus, dizziness has
been admitted to the cardiology department. Objectively: AP- 180/110 mm Hg, heart rate
- 95/min. Radiographically, there is a stenosis of one of the renal arteries. Hypertensive
condition in this patient has been caused by the activation of the following system:
A. Renin-angiotensin
B. Hemostatic
C. Sympathoadrenal
D. Kinin
E. Immune

48
9. A sportsman spontaneously held breath for 40 seconds, which resulted in an increase
in heart rate and systemic arterial pressure. Changes of these indicators are due to
activation of the following regulatory mechanisms:
A. Unconditioned sympathetic reflexes
B. Unconditioned parasympathetic reflexes
C. Conditioned sympathetic reflexes
D. Conditioned parasympathetic reflexes
E. –

10. A 43-year-old-patient has arterial hypertension caused by an increase in cardiac


output and general peripheral resistance. Specify the variant of hemodynamic
development of arterial hypertension in the given case:
A. Eukinetic
B. Hyperkinetic
C. Hypokinetic
D. Combined
E. –

11. A 16-year-old female patient has fainted after quickly changing her body position
from horizontal to vertical one. Which process from the ones listed below has caused the
loss of consciousness in the first place?
A. Decreasing venous return
B. Increasing venous return
C. Increasing central venous pressure
D. Decreasing oncotic pressure of blood plasma
E. Increasing arterial pressure

12. Pulmonary edema developed in a patient with hypertonic crisis. What is the main
factor in the pathogenesis of his state?
A. Increase of hydrostatic pressure in the capillary of the lungs.
B. Permeability increase of the vessels of pulmonary circulation
C. Increase of hydrostatic pressure in the capillary of the lungs.
D. Resistibility increase of the lung vessels
E. Decrease of oncotic pressure of blood plasma.

13. At examination a patient’s arterial pressure is 190/100 hg. What factors leads to
increase of arterial pressure?
A. Spasm of resistance vessels
B. Increase of venous recurrence
C. toxygenic dilation of cardiac muscle
D. Aler-Lilestrand reflex
E. Kitaevs reflex.

14. A patient has constant high arterial pressure - 160/110 mm of hg. Periodically it may
rise up to 220-240/120-130 mm of hg. What type of hypertension has this patient?
A. Adrenal
B. Cerebral
C. Renal
49
D. Psycho emotional
E. Essential

15. A patient with ischemic heart disease had a sudden severe attack of angina pectoris:
the face is pale, cold, damp skin, AP-70/50 mm Hg, extrasystolia. Myocardial infarction
and cardiogenic shock were diagnosed. Name the leading link of pathogenesis?
A. Decrease of minute blood volume
B. Pain syndrome
C. Toxaemia
D. Extrasystolia
E. Decrease of minute blood volume

16. Choose in what form below mentioned states that cause of cardiac insufficiency
heterometric mechanism of compensation (Frank-Starling) takes place:
A. Aortic incompetence
B. Hypertension of pulmonary circulation
C. Stenosis of mitral orifice
D. Stenosis of aortic osteum
E. Aortic incompetence

17. A 24-year-old female patient was admitted to the hospital with complaints of
headache, pain in kidney area, and general weakness. She was suffered from tonsillitis
one month before. At examination patient has BP of 180/110 mmHg; in patient’s blood
analysis erythrocytes – 3.1*1012/L, leucocytes – 12.6*109/L, ESR – 28 mm/hour; in
patient’s urinalysis – marked proteinuria, hematuria, leukocyturia. What is the
mechanism of hypertension development?
A. Renal
B. Reflexogenic
C. Aldosteron-induced
D. Renovascular
E. Mineralocorticoid-induced

18. The first link in atherosclerosis development is:


A. Disorders of intactness of arterial wall
B. Degenerative-proliferative changes of internal layer of arteries
C. Excessive depositions of blood plasma lipoproteins in internal layer of arteries
D. Formation of fibrous plaque on internal layer of arteries
E. Slowing of blood flow

19. A patient with myocardial infarction has a mark paleness of skin, oliguria, AP 100/90
mm Hg, and pulse 100 beats/min. What compensative mechanism maintains relative high
level of AP?
A. Centralization of blood circulation
B. Hypoperfusion of the lungs
C. Hypokalemia
D. Increase of the level of vasodilators in blood
E. Secondary hyperaldosteronism

50
20. A 58-year-old patient suffers from the cerebral atherosclerosis. Examination revealed
hyperlipoidemia. What class of lipoproteins will most probably show increase in
concentration in this patient’s blood serum?
A. Low-density lipoproteins
B. High-density lipoproteins
C. Fatty acid complexes with albumins
D. Chylomicrons
E. Cholesterol

21. After a psychic trauma a woman developed periodical increases in her blood pressure
accompanied by headache, palpitations, and general weakness. What mechanism of
hypertension development does this woman have?
A. Increased arteriolar tone
B. Increased circulating blood volume
C. Decreased cardiac output
D. Tachycardia
E. Venoconstriction

22.Atria of a test animal were superdistended with blood, which resulted in decreased
reabsorption of N a+ and water in renal tubules. This can be explained by the effect of the
following factor on the kidneys:
A. Natriuretic hormone
B. Aldosterone
C. Renin
D. Angiotensin
E. Vasopressin

23. In the process of an experiment, vascular resistance to the blood flow was measured
in the different areas of circulatory system. The highest resistance was detected in the:
A. Arterioles
B. Arteries
C. Capillaries
D. Venules
E. Veins

24. In human organism significant blood loss leads to decreased blood pressure,
tachycardia, and weakness. Eventually the sensation of thirst appears. What hormone
participates in the development of this sensation?
A. Angiotensin 2
B. Cortisol
C. Serotonin
D. Dopamine
E. Adrenalin

25. During ultrasound a patient with atherosclerosis was diagnosed with bilateral stenosis
of the renal arteries. Specify the bioactive substance that is the key pathogenetic link in
the development of arterial hypertension in this case:
A. Renin
51
B. Adrenaline
C. Vasopressin
D. Cortisol
E. Thyroxin

26.A67-year-oldmanconsumeseggs,pork fat,butter,milkandmeat.Bloodtestresults:
cholesterol - 12,3 mmol/l, total lipids - 8,2 g/l, increased low-density lipoprotein fraction
(LDL). What type of hyperlipoproteinemia is observed in the patient?
A. Hyperlipoproteinemiatype IIa
B. Hyperlipoproteinemiatype I
C. Hyperlipoproteinemiatype IIb
D. Hyperlipoproteinemiatype IV
E. Cholesterol,hyperlipoproteinemia

27. A patient has insufficient blood supply to the kidneys, which caused the development
of pressor effect due to the constriction of arterial resistance vessels. This is the result of
the vessels being greately affected by the following substance:
A. Angiotensin II
B. Angiotensinogen
C. Renin
D. Catecholamines
E. Norepinephrine

28. Atria of an experimental animal were superdistended with blood, which resulted
indecreasedreabsorptionofNa+ andwater in renal tubules. This can be explained by the
influence of the following factor on kidneys:
A. Natriuretic hormone
B. Aldosterone
C. Renin
D. Angiotensin
E. Vasopressin

Pathophysiology of external breathing

1. X-ray examination discovered lungs emphysema in the patient. What is the reason of
short breath development in this case?
A. Decreased lungs elasticity
B. Increased lungs elasticity
C. Inhibition of respiratory center
D. Excitation of respiratory center
E. Decreasing of alveoli receptors sensitivity

2.A 62-year-old patient was admitted to the neurological department due to cerebral
hemorrage. His condition is grave. There is evident progression of deep and frequent
breath that turnes into reduction to apnoea and the cycle repeats. What respiration type
has developed in the patient?
A. Cheyne-Stockes respiration
B. Kussmaul respiration
52
C. Biot’s respiration
D. Gasping respiration
E. Apneustic respiration

3. A patient after pathological process has a thickened alveolar membrane. The direct
consequence of the process will be the reduction of:
A. Diffuse lung capacity
B. Oxygen capacity of blood
C. Minute respiratory capacity
D. Alveolar lung ventilation
E. Reserve expiratiory capacity

4. While having the dinner the child choked and aspirated the food. Heavy cough has
started, skin and mucose are cyanotic, pulse is rapid, respiration is infrequent, expiration
is prolonged. What disorder of the external respiration has the child?
A. Stage of expiratory dyspnea on asphyxia
B. Stage of inspiratory dyspnea on asphyxia
C. Stenotic respiration
D. Alternating respiration
E. Biot’s respiration

5. A patient with thrombophlebitis of lower extremities had got chest pains, blood
spitting, growing respiratory failure that caused his death. Autopsy revealed multiple
pulmonary infarctions. What is the most probable reason of their development?
A. Pulmonary artery embolism
B. Pulmonary artery thrombosis
C. Bronchial artery thrombosis
D. Bronchial artery embolism
E. Pulmonary venous thrombosis

6. A group of mountain climbers went through the blood analysis at the height of 3000 m.
It revealed decrease of HCO3 to 15 micromole/l (standard is 22- 26 micromole/l). What
is the mechanism of HCO3 decrease?
A. Hyperventilation
B. Intensification of acidogenesis
C. Hypoventilation
D. Decrease of ammoniogenesis
E. Decrease of bicarbonate reabsorption in kidneys

7. Examination of a miner revealed pulmonary fibrosis accompanied by disturbance of


alveolar ventilation. What is the main mechanism of this disturbance?
A. Limitation of respiratory surface of lungs
B. Constriction of superior respiratory tracts
C. Disturbance of neural respiration control
D. Limitation of breast mobility
E. Bronchi spasm

53
8. A patient was admitted to the hospital with an asphyxia attack provoked by a spasm of
smooth muscles of the respiratory tracts. This attack was mainly caused by alterations in
the following parts of the airways:
A. Small bronchi
B. Median bronchi
C. Large bronchi
D. Terminal bronchioles
E. Respiratory part

9. While eating a child choked on food and aspirated it. The child has severe cough,
cyanotic skin and mucous membranes, rapid pulse, infrequent respiration, prolonged
expiration. The child has developed the following disorder of the external respiration:
A. Expiratory dyspnea under asphyxia
B. Inspiratory dyspnea under asphyxia
C. Stenotic respiration
D. Alternating respiration
E. Biot’s respiration

10. A 12-year-old adolescent suffering from bronchial asthma has a severe attack of
asthma: he presents with marked expiratory dyspnea, skin pallor. What type of alveolar
ventilation disorder is observed?
A. Obstructive
B. Restrictive
C. Thoracodiaphragmatic
D. Central
E. Neuromuscular

11. A patient staying in the pulmonological department was diagnosed with pulmonary
emphysema accompanied by reduced elasticity of pulmonary tissue. What type of
respiration is observed?
A. Expiratory dyspnea
B. Inspiratory dyspnea
C. Superficial respiration
D. Infrequent respiration
E. Periodic respiration

12. An unconscious young man with signs of morphine poisoning entered admission
office. His respiration is shallow and infrequent which is caused by inhibition of
respiratory centre. What type of respiratory failure is it?
A. Ventilative dysregulatory
B. Ventilative obstructive
C. Ventilative restrictive
D. Perfusive
E. Diffusive

13. Voluntary breath holding caused increase of respiration depth and frequency. The
main factor stimulating these changes of external respiration is:
A. Increased tension of CO2 in blood
54
B. Increased tension of O2 in blood
C. Decreased tension of O2 in blood
D. Decreased tension of CO2 in blood
E. Decreased concentration of H+ in blood

14. A patient with marked pneumofibrosis that developed after infiltrating pulmonary
tuberculosis has been diagnosed with respiratory failure. What is its pathogenetic type?
A. Restrictive
B. Obstructive
C. Dysregulatory
D. Reflex
E. Apneistic

15. A 23-year-old patient has been admitted to a hospital with a cranio-cerebral injury.
The patient is in a grave condition. Respiration is characterized by prolonged convulsive
inspiration followed by a short expiration. What kind of respiration is it typical for?
A. Apneustic
B. Gasping breath
C. Kussmaul’s
D. Cheyne-Stokes
E. Biot’s

16. A patient with bronchial asthma has developed acute respiratory failure. What kind
of respiratory failure occurs in this case?
A. Obstructive disturbance of alveolar ventilation
B. Restrictive ventilatory defect
C. Perfusion
D. Diffusion
E. Dysregulation of alveolar ventilation

17. Electrophoretic study of a blood serum sample, taken from the patient with
pneumonia, revealed an increase in one of the protein fractions. Specify this fraction:
A. γ-globulins
B. Albumins
C. α1-globulins
D. α2-globulins
E. β-globulins

18. Analysis of the experimental spirogram of a 55-year-old person revealed a decrease in


tidal volume and respiratory amplitude compared to the situation of ten years ago. The
change in these indicators is caused by:
A. Decreased force of respiratory muscle contraction
B. Gas composition of the air
C. Physical build of a person
D. Height of a person
E. Body mass of a person

55
19. A patient has increased thickness of alveolar-capillary membrane caused by a
pathologic process. The direct consequence will be reduction of the following value:
A. Diffusing lung capacity
B. Oxygen capacity of blood
C. Respiratory minute volume
D. Alveolar ventilation of lungs
E. Expiratory reserve volume

20. When studying the signs of pulmonary ventilation, reduction of forced expiratory
volume has been detected. What is the likely cause of this phenomenon?
A. Obstructive pulmonary disease
B. Increase of respiratory volume
C. Increase of inspiratory reserve volume
D. Increase of pulmonary residual volume
E. Increase of functional residual lung capacity

21. A 28-year-old patient undergoing treatment in the pulmonological department has


been diagnosed with pulmonary emphysema caused by splitting of alveolar septum by
tissular tripsin. The disease is caused by the congenital deficiency of the following
protein:
A. α1-proteinase inhibitor
B. α2-macroglobulin
C. Cryoglobulin
D. Haptoglobin
E. Transferrin

22. A patient demonstrates sharp decrease of pulmonary surfactant activity. This condi-
tion can result in:
A. Alveolar tendency to recede
B. Decreased airways resistance
C. Decreased work of expiratory muscles
D. Increased pulmonary ventilation
E. Hyperoxemia

23.When studying the pulmonary ventilation values, the reduction of forced expi- ratory
volume has been detected. What is the likely cause of this phenomenon?
A. Obstructive pulmonary disease
B. Increase of respiratory volume
C. Increase of inspiratory reserve volume
D. Increase of pulmonary residual volume
E. Increase of functional residual lung capacity

24. Miners’ work at the coal-face often leads to development of anthracosis. What type of
respiratory failure arises along with this disease?
A. Restrictive
B. Obstructive
C. Dysregulatory
D. Thoracic
56
E. Diaphragmatic

25. During ascent into mountains a person develops increased respiration rate and rapid
heart rate. What is the cause of these changes?
A. Decreaseof O2 partial pressure
B. Increase of CO2 partial pressure
C. Increase of blood pH
D. Increase of nitrogen content in air
E. Increase of air humidity

26. A patient, who has been suffering from bronchial asthma for a long time,developed
acute respiratory failure. What is the main mechanism of pathology development in this
case?
A. Obstructive disorders of pulmonary ventilation
B. Restrictive disorders of pulmonary ventilation
C. Pulmonary blood supply disturbance
D. Pulmonary enzyme system disturbance
E. Decreased elasticity of the pulmonary tissue

27. A 30-year-old man had suffered a thoracic trauma in a traffic accident,which resulted
in disturbance of external respiration. What ventilatory failure can be observed in this
case?
A. Extrapulmonary restrictive
B. Pulmonary restrictive
C. Obstructive
D. Dysregulatory
E. Mixed type

28. During an experiment aimed as study of respiration regulation processes the


peripheral chemoreceptors of test animals were stimulated, which resulted in changed
respiratory rate and depth. Where are these receptors localized?
A. Aortic arch, carotid sinus
B. Capillary bed, aortic arch, carotid sinus
C. Capillary bed, aortic arch
D. Capillary bed, carotid sinus
E. Atria, carotid sinus

29. Premature babies often develop respiratory distress syndrome. This pathology is
caused by the deficiency of a certain component of the blood–air barrier. Name this
component:
A. Surfactant
B. Capillary endothelium
C. Endothelial basement membrane
D. Alveolar basement membrane
E. Alveolocytes

30. A patient demonstrates sharp decrease of pulmonary surfactant activity.Thiscondition


can result in:
57
A. Alveolartendency to recede
B. Decreased airwaysresistance
C. Decresed work of expiratorymuscles
D. Increased pulmonary ventilation
E. Hyperoxemia
31. An uconscious young man in the state of morphine intoxication has been delivered to
admission room.The patient’s respiration is slow and shallow due to suppression of the
respiratory center. What kind of respiratory failure occurred in this case?
A. Ventilatorydisregulation
B. Ventilatoryobstruction
C. Ventilatoryrestriction
D. Perfusion
E. Diffusion

32. A patient has a traumatic injury of sternocleidomastoid muscle. This has


resultedinadecreaseofthefollowingvalue:
A. Inspiratory reserve volume
B. Expiratoryreserve volume
C. Respiratory volume
D. Residual volume
E. Functional residual lung capacity

33. When studying the signs of pulmonary ventilation, reduction of forced expiratory
volume has been detected. What is the likelycause of this phenomenon?
A. Obstructive pulmonary disease
B. Increase of respiratoryvolume
C. Increase of inspiratoryreserve volume
D. Increase of pulmonary residual volume
E. Increase of functional residual lung capacity

34. During recording of a spirogram a patient calmly exhaled. How do we call the
volume of air remainingin the lungs?
A. Functional residual capacity
B. Pulmonary residual volume
C. Expiratoryreserve volume
D. Tidal volume
E. Vital capacity of lungs

35. A patient diagnosed with acute respiratory failure has been administered artificial
lung ventilation in the conditions of high partial oxygen pressure. This measure resulted
in aggravation of the patient’s condition and development of respiratory distress
syndrome. Name the likely cause of this complication:
A. Intense oxidation of lung surfactant
B. Inflammatory process
C. Fibrosis
D. Atelectasis
E. Pulmonary congestion

58
36. A patient has sustained a traumatic injury of the greater pectoral muscle. This resulted
in the decrease of:
A. Inspiratory reserve volume
B. Expiratory reserve volume
C. Tidal volume
D. Residual volume
E. Functional residual lung capacity

Pathophysiology of digestion

1. A 57-year-old patient was admitted to the gastroenterological department with


suspicion of Zollinger-Ellison syndrome because of rapid increase of gastrin level in the
blood serum. What the most probable disorder of the secretory function of the stomach
here?
A. Hyperacidity hypersecretion
B. Hyperacidity hyposecretion
C. Achylia
D. Hypoacidity hyposecretion
E. Hypoacidity hypersecretion

2. 2 years ago a patient underwent resection of pyloric part of stomach. He complains of


weakness, periodical dark shadows beneath his eyes, dyspnea. In blood: Hb - 70 g/l,
erythrocytes - 3, 0 · 1012/l, colour index - 0,7. What changes of erythrocytes in blood
smears are the most typical for this condition?
A. Microcytes
B. Megalocytes
C. Schizocytes
D. Ovalocytes
E. Macrocytes

3. A 3 year old child with symptoms of stomatitis, gingivitis and dermatitis of open skin
areas was delivered to a hospital. Examination revealed inherited disturbance of neutral
aminoacid transporting in the bowels. These symptoms were caused by the deficiency of
the following vitamin:
A. Niacin
B. Pantothenic acid
C. Vitamin A
D. Cobalamin
E. Biotin

4. A doctor recommends a patient with duodenal ulcer to drink cabbage and potato juice
after the therapy course. Which substances contained in these vegetables help to heal and
prevent the ulcers?
A. Vitamin U
B. Pantothenic acid
C. Vitamin C
D. Vitamin B1
E. Vitamin K
59
5. A coprological survey revealed lightcolored feces containing drops of neutral fat. The
most likely reason for this condition is the disorder of:
A. Bile inflow into the bowel
B. Gastric juice acidity
C. Pancreatic juice secretion
D. Intestinal juice secretion
E. Intestinal absorption

6. A 30 year old woman has subnormal concentration of enzymes in the pancreatic juice.
This might be caused by the hyposecretion of the following gastrointestinal hormone:
A. Cholecystokinin-pancreozymin
B. Somatostatin C. Secretin
C. Gastro-inhibiting peptide
D. Vaso-intestinal peptide
E. –

7. A 42 year old patient complains of pain in the epigastral area, vomiting; vomit masses
have the colour of "coffee-grounds", the patient has also melena. Anamnesis records
gastric ulcer. Blood formula: erythrocytes - 2, 8 · 1012/l, leukocytes - 8 · 109/l, Hb- 90
g/l. What complication is it?
A. Haemorrhage
B. Penetration
C. Perforation
D. Canceration
E. Pyloric stenosis

8. Blood test of a patient suffering from atrophic gastritis gave the following results:
RBCs - 2, 0 · 1012/l, Hb- 87 g/l, colour index - 1,3, WBCs - 4, 0 · 109/l, thrombocytes -
180 · 109/l. Anaemia migh have been caused by the following substance deficiency:
A. Vitamin B12
B. Vitamin A
C. Vitamin K
D. Iron
E. Zinc

9. A patient has a critical impairment of protein, fat and hydrocarbon digestion. Most
likely it has been caused by low secretion of the following digestive juice:
A. Pancreatic juice
B. Saliva
C. Gastric juice
D. Bile
E. Intestinal juice

10. Steatosis is caused by the accumulation of triacylglycerols in hepatocytes. One of the


mechanisms of this disease development is a decrease in the utilization of VLDL neutral
fat. What lipotropics prevent the development of steatosis?
A. Methionine, B6, B12
60
B. Arginine, B2, B3
C. Alanine, B1, P P
D. Valine, B3, B2
E. Isoleucine, B1, B2

11. After taking a fatty food a patient feels nausea, flaccidity, later the sign of
steathorrhea has appeared, cholesterol in the blood 9,2ml/c. The cause of this state is the
deficiency of:
A. Bile acids
B. Triglyceroides.
C. Chylomicrones.
D. Fatty acids.
E. Phopholipids.

12. A patient aged 45 had the diagnosis ulcer of the stomach. On examination of
secretory function of the stomach it was determined that the amount of basal secretion
was 100mole/hr, acidity of basal secretion -60mml/hr. What factors action contributes to
the hypersecretion in the stomach?
A. Gastrine.
B. Pancreatic polypeptide.
C. Somatostatin.
D. Glucagon.
E. Betaendorphin.

13. On laboratory examination increased amount of diastase in the urine and also a large
amount of undigested fat in stool were revealed in a patient female with complain of
circular character pain in epigastric area. What form of gastrointestinal tract pathology
are described signs typical for?
A. Acute pancreatitis.
B. Acute appendicitis.
C. Infectious gastritis.
D. Ulcerous disease of the stomach.
E. Acute pancreatitis.

14. A part of patient pancreas was resected. What kinds of product must be limited in his
diet?
A. Fatty and fried meal.
B. Fruits.
C. Not fatty boiled meat.
D. Sour milk product
E. Vegetables.

15. What enzyme deficiency is the cause of maldigestion of fats in the gastrointestinal
tract and increase of neutral fats in the feces?
A. Enterokinase.
B. Intestinal lipase.
C. Hepatic lipase,
D. Gastric lipase
61
E. Pancreatic juice.

16. A patient had been taking antibiotics of a wide spectrum of action for a long period of
time that caused decrease of appetite, nausea, and diarrhea with saprogenic smell. What is
the side effect of treatment?
A. Dysbacteriosis.
B. Allergic reaction.
C. Hepatotoxic action
D. Nephrotoxic action.
E. Direct irritative action

17. On examination of a patient suffering from acute pancreatitis increased amount of


chylomicrons was determined in the blood. What enzyme activity is sharply decreased in
this pathology?
A. Lipoprotein lipase.
B. Pancreatic lipase.
C. Pancreatic phospholipase.
D. Tissue triglyceride lipase.
E. Tissue diglyceride lipase.

18. The analysis of gastric juice of an elderly man who complained of unmotivated
weakness, sickness, absence of appetite showed achylia, achlorhydrea, and presence of
lactic acids and coagulated blood, decreased of pepsin secretion. What disease causes
such clinical-laboratory symptoms?
A. Cancer of the stomach.
B. Chronic gastritis.
C. Chronic pancreatitis.
D. Cavitary Maldigestion.
E. Acute gastritias

19. A man with chronic hepatitis has dyspeptic disorders: decrease of appetite, nausea,
unstable stool, and steatorrhea. What is the mechanism of dyspeptic disorders in hepatic
pathology?
A. Hypocholea
B. Hypoglycemia
C. Cholalemia
D. Intoxication
E. Hyperbilirubemia

20. Spasmodic pains in the abdomen and repeated diarrhoea with mucus appeared in a
healthy person 3-5 hours later after taking meals. This was preceded by nausea and
momentary vomiting, general weakness, loss of appetite. What is the most possible cause
of the desired symptoms?
A. Food intoxication
B. Chronic pancreatitis
C. Enterocolitis
D. Hyperacid state of the stomach
E. Chronic gastritis
62
21. A man who works at a storage battery plants complains of constant feeling of weight
and periodical spasmodic pains in the abdomen, constant retention of stool (not more
often than one time per three day). This is accompanied by frequent headaches, flaccidity,
absence of appetite, and bad taste in the mouth. What are the causes of these disorders?
A. Spastic lead colie with constipation
B. Hypoacid state of the stomach
C. Hyper acid state of the stomach
D. Chronic pancreatitis
E. Parietal maldigestion

22. Due to chronic gastritis a man has the impaired structure of the mucous membrane,
decreased indices of acid formation function of the stomach. The most essential negative
result of this will be the impairment of:
A. Protien digestion
B. Pancreatic juice secretion
C. Secretory function of small intestine
D. Evacuation of chyme into duodenum
E. Excretion of secretum

23. A patient underwent a surgery for excision of a cyst on pancreas. After this he
developed hemorrhagic syndrome with apparent disorder of blood coagulation.
Development of this complication can be explained by:
A. Activation of fibrinolytic system
B. Insufficient fibrin production
C. Reduced number of thrombocytes
D. Activation of anticoagulation system
E. Activation of Christmas factor

24. Stool test detects in the patients feces a large amount of undigested fats. This patient
is the most likely to have disturbed secretion of the following enzymes:
A. Pancreatic lipases
B. Pancreatic amylase
C. Pancreatic proteases
D. Bile lipase
E. Gastric protease

25.An experimental animal, a dog, received a weak solution of hydrochloric acid


through a tube inserted into the duodenum. Primarily it will result in increased secretion
of the following hormone:
A. Secretin
B. Gastrin
C. Histamine
D. Cholecystokinin
E. Neurotensin

26. Various types of muscle contractions occurring in the alimentary canal of a test
animal were studied and their different functional purposes were determined. It was noted
63
that only one type of motor activity occurred in the circular and longitudinal muscles.
Name this motor activity:
A. Peristalsis
B. Mastication
C. Nonpropulsive segmental activity
D. Pendular movements of intestine
E. Tonic contraction of sphincters

27. The patient’s saliva has been tested for antibacterial activity. What saliva component
has antibacterial properties?
A. Lysozyme
B. Amylase
C. Ceruloplasmin
D. Parotin
E. Cholesterol

28. After examining a patient a doctor recommended him to exclude rich meat and
vegetable broths, spices, and smoked products from the diet, sincethe patient was found
to have:
A. Increased secretion of hydrochloric acid by the stomach glands
B. Reduced secretion of hydrochloric acid by the stomach glands
C. Reduced motility of the gastrointestinal tract
D. Reduced salivation
E. Biliary dyskinesia

29. A patient is diagnosed with pancreatitis. Starch decomposition disturbance occurs in


the patient’s intestine due to deficiency of the following pancreatic enzyme:
A. Amylase
B. Tripsin
C. Chymotrypsin
D. Lipase
E. Carboxypeptidase

30. A 42-year-old patient complains of pain in the epigastral area, vomiting; vomit
masses have the color of coffee-grounds; thepatientsuffersfrom melena.Anamnesis
recordsgastriculcerdisease.Bloodformula: erythrocytes - 2,8 · 1012/l, leukocytes 8 · 109/l,
Hb- 90 g/l. What complication is it?
A. Hemorrhage
B. Penetration
C. Perforation
D. Canceration
E. Pyloric stenosis

31. A 35-year-old man with peptic ulcer disease has undergone antrectomy. After the
surgery secretion of the following gastrointestinal hormone will be disrupted the most:
A. Gastrin
B. Histamine
C. Secretin
64
D. Cholecystokinin
E. Neurotensin
32. A patient with gastric juice hypersecretion has been recommended to exclude from
the diet rich broths and vegetable infused water. A doctor recommended it, because these
food products stimulate production of the following hormone:
A. Gastrin
B. Secretin
C. Cholecystokinin
D. Somatostatin
E. Neurotensin

Pathophysiology of the liver

1. A patient complains of frequent diarrheas, especially after consumption of fattening


food, and of body weight loss. Laboratory examination revealed steatorrhea; hypocholic
feces. What can be the cause of this condition?
A. Obturation of biliary tracts
B. Mucous membrane inflammation of small intestine
C. Lack of pancreatic lipase
D. Lack of pancreatic phospholipase
E. Unbalanced diet

2. After consumption of rich food a patient has nausea and heartburn, steatorrhea. This
condition might be caused by:
A. Bile acid deficiency
B. Increased lipase secretion
C. Disturbed tripsin synthesis
D. Amylase deficiency
E. Disturbed phospholipase synthesis

3. A patient has yellow skin colour, dark urine, dark-yellow feces. What substance will
have strengthened concentration in the blood serum?
A. Unconjugated bilirubin
B. Conjugated bilirubin
C. Mesobilirubin
D. Verdoglobin
E. Biliverdin

4. A patient is ill with hepatocirrhosis. State of antitoxic liver function can be


characterized by examination of the following substance excreted by urine:
A. Hippuric acid
B. Ammonium salts
C. Creatinine
D. Uric acid
E. Amino acids

65
5. A patient underwent an operation on account of gall bladder excision that resulted in
obstruction of Ca absorption through the bowels wall. What vitamin will stimulate this
process?
A. D3
B. PP
C. C
D. B12
E. K

6. A patient being treated for viral hepatitis type B got symptoms of hepatic
insufficiency. What blood changes indicative of protein metabolism disorder will be
observed in this case?
A. Absolute hypoalbuminemia
B. Absolute hyperalbuminemia
C. Absolute hyperfibrinogenemia
D. Proteinic blood composition is unchanged
E. Absolute hyperglobulinemia

7. A 46 year old woman suffering from chololithiasis developed jaundice. Her urine
became dark-yellow and feces became colourless. Blood serum will have the highest
concentration of the following substance:
A. Conjugated bilirubin
B. Unconjugated bilirubin
C. Biliverdin
D. Mesobilirubin
E. Urobilinogen

8. Hepatitis has led to the development of hepatic failure. Mechanism of edemata


formation is activated by the impairment of the following liver function:
A. Protein-synthetic
B. Barrier
C. Chologenetic
D. Antitoxic
E. Glycogen-synthetic

9. Blood analysis of a patient with jaundice reveals conjugated bilirubinemia, increased


concentration of bile acids. There is no stercobilinogen in urine. What type of jaundice is
it?
A. Obstructive jaundice
B. Hepatocellular jaundice
C. Parenchymatous jaundice
D. Hemolytic jaundice
E. Cythemolytic jaundice

10. A patient presents with icteric of skin, scleras and mucous membranes. Blood plasma
the total bilirubin is increased, stercobilin is increased in feces, urobilin is increased in
urine. What type of jaundice is it?
A. Haemolytic
66
B. Gilbert’s disease
C. Parenchymatous
D. Obturational
E. Cholestatic

11. After severe viral hepatitis a 4 year old boy presents with vomiting, occasional loss of
consciousness, convulsions. Blood test revealed hyperammoniemia. Such condition is
caused by a disorder of the following biochemical hepatic process:
A. Disorder of ammonia neutralization
B. Disorder of biogenic amines neutralization
C. Protein synthesis inhibition
D. Activation of amino acid decarboxylation
E. Inhibition of transamination enzymes

12. A patient with jaundice has high total bilirubin that is mainly indirect (unconjugated),
high concentration of stercobilin in the stool and urine. The level of direct (conjugated)
bilirubin in the blood plasma is normal. What kind of jaundice can you think of?
A. Hemolytic
B. Parenchymal (hepatic)
C. Mechanical
D. Neonatal jaundice
E. Gilbert’s disease

13. Feces of a patient contain high amount of undissociated fats and have grayish-white
color. Specify the cause of this phenomenon:
A. Obturation of bile duct
B. Hypoactivation of pepsin by hydrochloric acid
C. Hypovitaminosis
D. Enteritis
E. Irritation of intestinal epithelium

14. In inflammatory process colloidal properties of bile are impaired in gall bladder and
this results in the formation of gall stones. What substance crystallization is the main
cause of the formation?
A. Cholesterol
B. Urate
C. Chloride
D. Oxalate
E. Phosphate

15. Flabby contraction of gall bladder was revealed in a woman aged 55 after introducing
some of vegetable oil into duodenum. What hormone insufficiency with such state?
A. Cholecystokinin
B. Enterogastrin
C. VIP
D. Pancreozymin
E. Gastrin

67
16.A patient aged 25 has a diagnosis of chronic hepatitis. A patient has lost 10 kg of his
body weight for 2 months. Objectively: the skin is dry, desquamative, and pale with
yellowish color, small punctate hemorrhages on the skin, stomatorrhagia. The impairment
of what hepatic function do petechial hemorrhage and stomatorrhagia prove?
A. Albumin synthetic
B. ekromogenic
C. Detoxicative
D. Depositing
E. Glycogen synthetic

17. On examination bile congestion in the liver and cholelitiasis were revealed in a
patient. Point out the main component of cholelitiasis in this state:
A. Cholesterol
B. Triglycerides
C. Protein
D. Calcium bilirubinate
E. Mineral salts

18. Residual nitrogen and urea were determined in the patient’s blood analysis. The
disease of what organ is characterized by this analysis?
A. Liver
B. Kidneys
C. Stomach
D. Heart
E. Intestine

19. A patient complains of general weakness, boring pain in the abdomen, bad appetite,
suspicion on jaundice. Blood serum contains 77.3 mcml/L of total bilirubin and 70.76
mcml/L of conjugative bilirubin. What is the most possible type of jaundice?
A. Mechanical jaundice
B. Acute hepatitis
C. Hepatic cirrhosis
D. Parenchymatous jaundice
E. Hemolytic jaundice

20. In 70’s the scientists determined that the cause of severe jaundice in newborns was
the impairment of connection of bilirubin in hepatocytes. What substance is used for the
formation of conjugate?
A. Glucuronic acid
B. Pyruvic acid
C. Uric acid
D. Sulphuric acid
E. Lactic acid

21. A patient suffers from hepatic cirrhosis. What substance excreted in urine should be
analyzed to characterize the antitoxic function of liver?
Hippuric acid
Ammonium salts
68
Creatinine
Uric acid
Amino acids
22. Patients with bile duct obstruction typically present with inhibited blood clotting and
develop hemorrhages due to insufficient assimilation of vitamin:
A. K
B. A
C. D
D. E
E. C

23. Encephalopathy has developed in a child with hemolytic disease of the newborn.
What substance had increased in the child’s blood, resulting in damage to the CNS?
A. Unconjugated bilirubin
B. Bilirubin-albumin complex
C. Bilirubin glucuronide
D. Verdohemoglobin
E. Bile acids

24. Detoxification of bilirubin occurs in the membranes of endoplasmic reticulum of


hepatocytes. Bilirubin is secreted by hepatocytes into bile for the most part as:
A. Bilirubin diglucuronide
B. Unconjugated bilirubin
C. Bilirubin monoglucuronide
D. Indirect reacting bilirubin
E. -

25. A 72-year-old man with hepatocirrhosis developed hepatic coma. Its development is
caused by the substances that are being neutralized in the liver, entering into general
circulation through portacaval shunts (portal hypertension syndrome) and necrosis of
hepatic cells. What type of hepatic coma is characterized by these presentations?
A. Mixed
B. Parenchymatous
C. Shunt
D. Hepatocellular
E. Ketoacidotic

26. A patient came to the doctor with complaints of general weakness and sleep
disturbances. Objectively the patient’s skin is yellow. In blood there is increased
concentration of direct bilirubin and bile acids. Acholic stool is observed. What condition
can be characterized by these changes?
A. Mechanical jaundice
B. Hemolytic jaundice
C. Parenchymatous jaundice
D. Familial nonhemolytic (Gilbert’s) syndrome
E. Chronic cholecystitis

69
27. A patient presents with steatorrhea. This disorder can be linked to disturbed supply of
the intestine with the following substances:
A. Bile acids
B. Carbohydrates
C. Tripsin
D. Chymotrypsin
E. Amylase

28. An unconscious patient was delivered by ambulance to the hospital. On objective


examination the patient was found to have noreflexes,periodicalconvulsions,irregular
breathing.Afterlaboratoryexaminationthe patient was diagnosed with hepatic coma.
Disorders of the central nervous system develop due to the accumulation of the
followingmetabolite:
A. Ammonia
B. Urea
C. Glutamine
D. Bilirubin
E. Histamine

29. A patient has been admitted to the contagious isolation ward with signs of
jaundicecausedby hepatitisvirus.Which of thesymptomsgivenbelowisstrictlyspecific for
hepatocellularjaundice?
A. Increase of ALT, AST level
B. Hyperbilirubinemia
C. Bilirubinuria
D. Cholemia
E. Urobilinuria

30. A 50-year-old man, who has been suffering for a long time from viral hepatitis,
developed mental impairments, impairments of consciousness, and motor disturbances
(tremor, ataxia, etc.). What is the mechanism of such condition?
A. Decreased detoxification function of the liver
B. Insufficient phagocytic function of stellate macrophages
C. Decreased synthesis of albumins and globulins in the liver
D. Disturbed lipid exchange in the liver
E. Alterations in the lipid composition of blood

31. Roentgenologically confirmed obstruction of common bile duct resulted in preventing


bile from inflowing to the duodenum. What process is likely to be disturbed?
A. Fat emulgation
B. Protein absorption
C. Carbohydrate hydrolysis
D. Hydrochloricacidsecretioninstomach
E. Salivation inhibition

32. Upon toxic damage of hepatic cells resulting in disruption of liver function the
patientdevelopededemas.Whatchangesof blood plasma are the main cause of edema
development?
70
A. Decreaseof albumin content
B. Increase of globulin conten
C. Decrease of fibrinogen conten
D. Increase of albuminconten
E. Decrease of globulinconten

33. A 15-year-old boy has been diagnosed with acute viral hepatitis. What blood value
should be determined to confirm acute affectionof hepatic cells?
A. Aminotransferaseactivity(AST, ALT)
B. Unconjugated and conjugated bilirubin content
C. Erythrocytes sedimentationrate (ESR)
D. Cholesterol content
E. Protein fraction content

34. A 16-year-old adolescent is diagnosed with hereditary UDP (uridine diphosphate)


glucuronyltransferase deficiency. Laboratory tests revealed hyperbilirubinemia caused
mostly by increased blood content of the following substance:
A. Unconjugated bilirubin
B. Conjugated bilirubin
C. Urobilinogen
D. Stercobilinogen
E. Biliverdine

35. Cholesterol content in blood serum of a 12-year-old boy is 25 mmol/l. Anamnesis


states hereditary familial hypercholesterolemia caused by synthesis disruption of
receptor-relatedproteins for:
A. Low-density lipoproteins
B. High-densitylipoproteins
C. Chylomicrons
D. Very low-densitylipoproteins
E. Middle-densitylipoproteins

36. For several days a 55-year-old woman has been suffering from pain attacks in the
right upper quadrant after eating fat food.Visually, there is yellowness of sclera and skin.
The patient has acholic stool, beer-colored urine. What substance present in the patient’s
urine causes its dark color?
A. Conjugated bilirubin
B. Ketone bodies
C. Unconjugated bilrubin
D. Stercobilin
E.Bilirubin glucuronides

37. Roentgenologically confirmed an obstruction of common bile duct that prevents bile
from inflowing to the duodenum. What process is likely to be disturbed?
A. Fat emulgation
B. Protein absorption
C. Carbohydrate hydrolysis
D. Hydrochloric acid secretion in stomach
71
E. Salivation inhibition

38. Specify the type of jaundice, during which there is no direct bilirubin in
blood, and urine urobilinogen level is high:
A. Suprahepatic
B. Hepatic
C. Subhepatic
D. Mechanical
E.

Pathophysiology of the kidneys

1. A 16 year-old patient got numerous traumas in automobile accident. Now the patient is
having a shock. АP - 80/60 mm Hg., daily urine volume 60-80 ml. What pathogenic
mechanism leads to kidneys function violation?
A. Decreased hydrostatic pressure in glomerular capillaries
B. Increased osmotic pressure in glomerular capillaries
C. Increased pressure in Bowman’s capsule
D. Increased vasopressin blood concentration
E. Trauma of the urinary bladder

2. Chronic glomerulonephritis was diagnosed in a 34-year-old patient 3 years ago. Edema


has developed within the last 6 months. What caused the edema?
A. Proteinuria
B. Hyperproduction of vasopressin
C. Liver disfunction of protein formation
D. Hyperosmolarity of plasma
E. Hyperaldosteronism

3. A 58-year-old patient with acute cardiac insufficiency has decreased volume of daily
urine - oliguria. What is the mechanism of this phenomenon?
A. Decreased glomerular filtration
B. Decreased number of functioning glomerules
C. Drop of oncotic blood pressure
D. Rise of hydrostatic blood pressure in capillars
E. Reduced permeamility of renal filter

4. Violation of safety rules resulted in calomel intoxication. Two days later the daily
diuresis was 620 ml. A patient experienced headache, vomiting, convulsions, dyspnea,
moist rales in lungs. What pathology is it?
A. Acute renal insufficiency
B. Chronic renal insufficiency
C. Uraemic coma
D. Glomerulonephritis
E. Pyelonephritis

72
5.A 65 year old man suffering from gout complains of kidney pain. Ultrasound
examination revealed renal calculi. The most probable cause of calculi formation is the
strengthened concentration of the following substance:
A. Uric acid
B. Cholesterol
C. Bilirubin
D. Urea
E. Cystine

6. A patient with nephrotic syndrome has massive edemata of his face and limbs. What is
the leading pathogenetic mechanism of edemata development?
A. Drop of oncotic blood pressure
B. Increase of vascular permeability
C. Rise of hydrodynamic blood pressure
D. Lymphostasis
E. Increase of lymph outflow

7. A patient with massive burns developed acute renal insufficiency characterized by a


significant and rapid deceleration of glomerular filtration. What is the mechanism of its
development?
A. Reduction of renal blood flow
B. Damage of glomerular filter
C. Reduction of functioning nephron number
D. Rise of pressure of tubular fluid
E. Renal artery embolism

8. A 30 year old woman has face edemata. Examination revealed proteinuria (5,87 g/l),
hypoproteinemia, dysproteinemia, hyperlipidemia. What condition is the set of these
symptoms typical for?
A. Nephrotic syndrome
B. Nephritic syndrome
C. Chronic pyelonephritis
D. Acute renal failure
E. Chronic renal failure

9. Urine analysis of a 12-year-old boy reveals high concentration of all aliphatic


aminoacids with the highest excretion of cystine and cysteine. Ultrasound examination of
kidneys revealed concrements presence. What is the most likely pathology?
A. Cystinuria
B. Alkaptonuria
C. Cystitis
D. Phenylketonuria
E. Hartnup disease

10. A 38-year-old male patient has been ill with systemic lupus erythematosus for three
years. He was diagnosed with diffuse renal affection accompanied by massive edemata
and expressive proteinuria. What is the most likely cause of proteinuria development?
A. Autoimmune renal affection
73
B. Aseptic renal affection
C. Ischemic renal affection
D. Urinary bladder inflammation
E. Urinary tracts inflammation

11. A patient with massive burns developed acute renal insufficiency characterized by a
significant and rapid deceleration of glomerular filtration. What is the mechanism of its
development?
A. Reduction of renal blood flow
B. Damage of glomerular filter
C. Reduction of functioning nephron number
D. Rise of pressure of tubular fluid
E. Renal artery embolism

12. A child has an acute renal failure. What biochemical factor found in saliva can
confirm this diagnosis?
A. Increase in urea concentration
B. Increase in glucose concentration
C. Decrease in glucose concentration
D. Increase in concentration of higher fatty acids
E. Decrease in nucleic acid concentration

13. A patient with a history of chronic glomerulonephritis presents with azotemia,


oliguria, hypo- and isosthenuria, proteinuria. What is the leading factor in the
pathogenesis of these symptoms development under chronic renal failure?
A. Mass decrease of active nephrons
B. Intensification of glomerular filtration
C. Tubular hyposecretion
D. Disturbed permeability of glomerular membranes
E. Intensification of sodium reabsorption

14. Atria of an experimental animal were superdistended by blood that resulted in


decreased reabsorption of Na+ and water in renal tubules. This can be explained by the
influence of the following factor upon kidneys:
A. Natriuretic hormone
B. Aldosterone
C. Renin
D. Angiotensin
E. Vasopressin

15. Due to the use of poor-quality measles vaccine for preventive vaccination, a 1-year-
old child developed an autoimmune renal injury. The urine was found to contain
macromolecular proteins. What process of urine formation was disturbed?
A. Filtration
B. Reabsorption
C. Secretion
D. Reabsorption and secretion
E. Secretion and filtration
74
16. Diabetic nephropathy with uremia has developed in a patient with pancreatic diabetes.
The velocity of glomerular filtration is 9 ml/min. What mechanism of a decrease in
glomerular filtration velocity and chronic renal failure development is most likely in the
case of this patient?
A. Reduction of active nephron mass
B. Decrease in systemic arterial pressure
C. Obstruction of nephron tubules with hyaline casts
D. Tissue acidosis
E. Arteriolar spasm

17. On the 6th day of treatment a patient with acute renal insufficiency developed
polyuria. Diuresis intensification at the beginning of polyuria stage of acute renal
insufficiency is caused by:
A. Renewal of filtration in nephrons
B. Volume expansion of circulating blood
C. Growth of natriuretic factor
D. Reduction of aldosteron content in plasma
E. Reduction of vasopressin content in plasma

18. A patient has severe nephropathy with a massive edematous syndrome, which
complicated bronchiectatic diseases. Laboratory investigations show massive proteinuria,
cylindruria, considerable decrease of protein content in the blood serum, hyperlipidemia,
hypokalemia and other deviations. What is the main link in the pathogenesis of the
patient’s edemas?
A. Decrease of oncotic blood pressure.
B. Increase of extracellular fluid pressure.
C. Increase of hydrostatic blood pressure.
D. Blockade of lymph drainage.
E. Hyperpermeability of microvessels

19. A man aged 32 has been ill with chronic glomerulonephritis for 4 years. He was
hospitalized with the signs of anasarca. AP-185/105 mm. Hg. Blood analysis shows: Hb-
110gm/l; erythrocytes - 2.6*1012/l; leucocytes - 9.5*109/l; residual nitrogen-32 mmol/l;
total protein-50 gm/l. What change points to glomerulonephritis with nephrotic
syndrome?
A. Hypoproteinemia
B. Arterial hupertension
C. Hyperzotemia
D. Hypoproteinemia
E. Leukocytosis

19. According to the results of glucose tolerance test, the patient has no disorder of
carbohydrate tolerance. Despite that, glucose is detected in the patients’s urine (5
mmol/l). The patient has been diagnosed with renal diabetes. What renal changes cause
glucosuria in this case?
A. Decreased activity of glucose reabsorption enzymes
B. Increased activity of glucose reabsorption enzymes
75
C. Exceeded glucose reabsorption threshold
D. Increased glucose secretion
E. Increased glucose filtration
20. A lab rat has subcutaneously received mercury(II) chloride in the amount of 5 mg/kg.
24 hours later the plasma creatinine concentration increased several times. What
mechanism of retention azotemia is observed in this case?
A. Decreased glomerular filtration
B. Increased creatinine production in the muscles
C. Increased creatinine reabsorption
D. Increased glomerular filtration
E. Increased creatinine production in the renal tubules

21. A 38-year-old man, who has been suffering from systemic lupus erythematosus for
3 years, developed di- ffuse renal lesions accompanied by massi- ve edemas, marked
proteinuria, hyperli- pidemia, and dysproteinemia. What is the most likely mechanism
of proteinuria develoment in this case?
A. Autoimmune damage to the nephrons
B. Inflammatory damage to the nephrons
C. Ischemic damage to the tubules
D. Increased blood proteins
E. Morbid affection of the urinary tracts

22. A 50-year-old inpatient during examination presents with glucosuria and blood
glucose of 3.0 mmol/L, which are the most likely to be caused by:
A. Renal disorder
B. Diabetes insipidus
C. Myxedema
D. Essential hypertension
E. Pellagra

23. A woman presents with edemas. In her urine there is a large amount of protein
excreted. What nephron segment is functionally disturbed in this case?
A. Renal corpuscle
B. Proximal convoluted tubule
C. Distal convoluted tubule
D. Descending limb of loop of Henle
E. Ascending limb of loop of Henle

24.A 55-year-old man was diagnosed with acute glomerulonephritis. Name the main
mechanism of anemia development in this case:
A. Decreased erythropoietin synthesis
B. Decreased glomerular filtration
C. Decreased synthesis of renal prostaglandins
D. Renal azotemia
E. Decreased tubular reabsorption

25. A 49-year-old man presents with facial edema, significant proteinuria, hypoprotei-
nemia, dysproteinemia, and hyperlipidemia. What provisional diagnosis can be made?
76
A. Nephrotic syndrome
B. Urolithiasis
C. Prostatitis
D. Pyelonephritis
E. Cystitis

26.Poisoningcausedbymercury(II) chloride (corrosive sublimate) occurred in the result of


safety rules violation.In 2 days the patient’s diurnal diuresis became 620 ml. The patient
developed headache, vomiting, convulsions, dyspnea; moist crackles are observedin
thelungs.Name thispathology:
A. Acute renal failure
B. Chronic renal failure
C. Uremiccoma
D. Glomerulonephritis
E. Pyelonephritis

27. A man presents with glomerular filtration rate of 180 ml/min., while norm is 125±25
ml/min. The likely cause of it is the decreased:
A. Plasma oncotic pressure
B. Effectivefiltrationpressure
C. Hydrostatic blood pressure in the glomerularcapillaries
D. Renal blood flow
E. Permeabilityof the renalfilter

28. A 50-year-old inpatient during examination presents with glucosuria and blood
glucose of 3,0 mmol/l, which are the most likelyto be caused by:
A. Renal disorder
B. Diabetesinsipidus
C. Myxedema
D. Essential hypertension
E. Pellagra

29. A patient with chronic renal failure presentswithreducedinulinclearanceof 60 ml/min.


The following renal function is disturbed:
A. Glomerular filtration
B. Tubular secretion
C. Reabsorption in the proximal tubular segment of the nephron
D. Reabsorption in the distal tubular segment of the nephron
E.Reabsorption inthetubulesofcollecting duct

30. A 19-year-old young man has been examined in a nephrological hospital. Increased
potassium content was detected in secondary urine of the patient. Such alterations are the
most likely to be caused by the increased secretion of the following hormone: A.
Aldosterone
B. Oxytocin
C. Adrenaline
D. Glucagon
E. Testosterone
77
31. A 30-year-old man with glomerulonephritis has developed nephrotic syndrome. What
symptom invariably accompanies nephrotic syndrome?
A. Proteinuria
B. Glucosuria
C. Low urine specific gravity
D. Anemia
E. Azotemia

32. Urine analysis has shown high levels of protein and erythrocytes in urine. This can be
caused by the following:
A. Renal filter permeability
B. Effectivefilter pressure
C. Hydrostatic blood pressure in glomerular capillaries
D. Hydrostatic primary urine pressure in capsule
E. Oncotic pressure of blood plasma

33.Poisoningcausedbymercury(II) chloride (corrosive sublimate) occurred in the result of


safety rules violation. In 2 days the patient’s diurnal diuresis was 620 ml. The patient
developed headache, vomiting, convulsions, dyspnea; moist crackles were observed inthe
lungs.Namethispathology:
A. Acute renal failure
B. Chronic renal failure
C. Uremiccoma
D. Glomerulonephritis
E. Pyelonephritis

34. A 12-year-old child developed nephritic syndrome (proteinuria, hematuria,


cylindruria) 2 weeks after a case of tonsillitis, which is a sign of affected glomerular
basement membrane in the kidneys. What mechanism is the most likely to cause the
basementmembranedamage?
A. Immune complex
B. Granulomatous
C. Antibody-mediated
D. Reaginic
E. Cytotoxic

35. After the transfusion of the concentrated red blood cells the patient developed
posttransfusion shock. What is the leading mechanism of the acute renal failure in this
case?
A. Glomerular filtration disorder
B. Tubular reabsorption disorder
C. Tubular secretion disorder
D. Urinary excretion disorder
E. Impairment of the renal incretory function

Pathophysiology of extreme states

78
1. Purulent endometritis developed in a woman after delivery. Treating with antibiotics
inhibitors of murein synthesis was ineffective. Wide spectrum bactericidal antibiotic was
administered to her. In 6 hours temperature rapidly increased up to 400C with shiver.
Muscle pains have appeared. BP dropped down to 70/40 mmHg. Oliguria has developed.
What is the main reason for the development of this condition?
A. Endotoxic shock
B. Toxic effect of preparation
C. Internal bleeding
D. Anaphylactic shock
E. Bacteremia

2. The patient has come to the hospital from the smelting workshop in the condition of
hyperthermia. What is the direct cause of loss of consciousness at the heat stroke?
A. Decreased brain blood supply
B. Arterial pressure drop
C. Increased water loss through sweating
D. Decrease of heart output
E. Dilatation of peripheral vessels

3. A 16 year-old patient got numerous traumas in automobile accident. Now the patient is
having a shock. АP - 80/60 mm Hg., daily urine volume 60-80 ml. What pathogenic
mechanism leads to kidneys function violation?
A. Decreased hydrostatic pressure in glomerular capillaries
B. Increased osmotic pressure in glomerular capillaries
C. Increased pressure in Bowman’s capsule
D. Increased vasopressin blood concentration
E. Trauma of the urinary bladder

4. A 26 year old man is in the torpid shock phase as a result of a car accident. In blood: 3,
2 · 109/l. What is the leading mechanism of leukopenia development?
A. Redistribution of leukocytes in bloodstream
B. Leikopoiesis inhibition
C. Disturbed going out of mature leukocytes from the marrow into the blood
D. Lysis of leukocytes in the blood-forming organs
E. Intensified elimination of leukocytes from the organism

5. As a result of a trauma a patient has developed traumatic shock that led to the
following disorders: AP is 140/90 mm Hg, Ps is 120 bpm. The patient is fussy, talkative,
pale. Such state relates to the following shock phase:
A. Erectile
B. Latent period
C. Terminal
D. Torpid
E. –

6. As a result of a road accident a driver has gotten a trauma. Now he is in shock


condition and presents with a decrease in daily diuresis down to 300 ml. What is the main
pathogenetic factor of such alteration in the diuresis?
79
A. Arterial pressure drop
B. Oncotic blood pressure drop
C. Increase in vascular permeability
D. Decrease in number of the functioning glomerules
E. Secondary hyperaldosteronism

7. After a road accident a victim has tachycardia, arterial blood pressure 130/90 mm Hg,
tachypnoe, the skin is pale and dry, excitation of central nervous system is observed.
What shock stage is the patient most likely in?
A. Erectile
B. Terminal
C. Torpid
D. Preshock (compensation stage)
E. Agony

8. A patient in comatose state has smell of acetone from his mouth. Content of glucose in
his blood plasma is of 18 mmol/L. What kind of coma is the most possible one in this
case?
A. Ketoacidemic
B. Hyperosmolar
C. Hypoglycemic
D. Toxic
E. Lactatacidemic

9. A patient with crushed muscular tissue was admitted to the traumatological


department. Which biochemical index of urine is increased in this case?
A. Creatinin
B. Mineral salts
C. Uric acid
D. General lipids
E. Glucose

10. A patient is drowsy, his conscious is depressed, and his reactions to irritants are
suspended. He has a pale dry and edematous skin, muscular fibrillations, mydriasis, and
Cheyne-Stocks’ respiration with ammonium scent from his mouth. Pericardial friction
sound was found at auscultation of the patient’s heart. What kind of coma has developed
in this patient?
A. Renal
B. Cerebral
C. Hyperosmolar
D. Hepatic
E. Apoplectic

11. A 35-year-old man has massive trauma of lower extremities without considerable
external bleeding. The victim is in exited condition. What component of pathogenesis of
traumatic shock is leading and needs immediate correction?
A. Pain
B. Internal bleeding
80
C. Intoxication
D. Disorder of organ functions
E. Internal loss of plasma

12. A patient with burn of 30% of body surface has decrease in BP to 75/20 mm Hg,
frequent, filiform pulse (110/min). What is the main factor of decrease in BP in burn
shock?
A. Plasmarrhea
B. Intoxication by decay products
C. Activation of sympathetic nervous system
D. Hypoproteinemia
E. Activation of parasympathetic nervous system

13. Spasm of resistant vessels develops in zones with alpha-adrenoreceptors in erectile


phase of shock. What hemodynamic changes will be observed in this case?
A. Decrease of blood flow speed
B. Centralization of blood flow
C. Bradycardia
D. Decrease of venous return
E. Systemic decrease of peripheral blood flow

14. A victim of the earthquake felt well just after he was extracted from under ruins, but
soon abrupt worsening of his condition developed. He lost consciousness, his BP was of
70/35 mmHg, and his pulse was of 90 per min; edema of previously ischemic tissues
increased quickly. Which is the main cause of patient’s condition worsening?
A. Systemic activation of proteolytic processes and PLO
B. Thrombosis of subcutaneous venous vessels
C. Dehydration
D. Increase in tonus of sympathetic nervous system
E. Disorders of renal functions

15. A patient was admitted to the hospital in severe state. He does not answer the
questions and does not react to pain stimuli. Patient’s BP is 50/10 mmHg, his pulse is 50
per min. What are the reasons for disorders of system hemodynamics at torpid phase of
shock?
A. Total decrease in peripheral vascular resistance
B. Elevation of venous return of blood
C. Increase of CBV
D. Rise of tonicity of sympathetic nervous system
E. Decrease in permeability of exchange vessels

16. Intravenous infusion of blood and blood substitutes does not lead to the stabilization
of BP in a patient with torpid phase of shock. After cessation of infusion patient’s blood
pressure decreases again. What does inefficacy of medical treatment in torpid phase of
shock connected to?
A. Systemic increase in vessel permeability
B. Hemodilution
C. Increase in velocity of blood flow
81
D. Tachycardia
E. Spasm of resistant vessels

17. Levels of plasma proteins are sharply increasing, and the number of alveolar
macrophages and lymphocytes is decreasing in broncho-alveolar contents of a patient
with shock. What is the mechanism of these phenomena appearance?
A. Increase in permeability of lung capillaries
B. Spasm of resistant vessels of the lungs
C. Increase in inhibitor systems of the lungs
D. Increase in pulmonary blood flow
E. Hyperventilation

18. A patient with torn away finger was admitted to the traumatological department. He
is fussy, verbose and pale. His pulse is 120 beats per minute, BP 140/90 mmHg. What are
the peculiarities of disorders of microcirculation and systemic hemodynamics in erectile
phase of shock?
A. Increase in systemic peripheral resistance
B. Systemic spasm of volume vessels
C. Decrease in venous return and volume of blood circulation
D. Increase in peripheral resistance in the brain, heart and lungs vessels
E. Hypoxia in zones with beta-adrenoreceptors

19. Hemodialysis was utilized for treatment for acute renal failure developed in a patient
as a result of traumatic shock. While carrying out the hemodialysis patient’s condition
became worse, aggregation of erythrocytes, leukocytes, and platelets occurred in
microvessels, and blood viscosity increased. What kind of disorders of microcirculation
developed in the patient?
A. Sludge syndrome
B. Disorders of vascular permeability
C. Extravascular disorders
D. Capillary-trophic insufficiency
E. Lymphatic system insufficiency

20. Tourniquet was applied to upper third of hip of wounded driver just at place of car
accident. The patient was admitted to a surgical department in satisfactory condition 3
hours later. Marked edema of hip tissues, frequent pulse, cold perspiration, and expressed
hypotension develop in the patient after removal of tourniquet. Which pathological
process develops in the patient?
A. Toxic shock
B. Anaphylactic shock
C. Collapse
D. Cardiogenic shock
E. Hemorrhagic shock

21. Daily diuresis amounts to 250 ml in a patient with anaphylactic shock. The patient
has moist rales in his lungs; his consciousness is intact. In patient’s blood acidosis reveals
with base deficiency of 14.5 mmol/L; plasma contents of potassium is 8.8 mmol/L and
urea is 48 mmol/L. How is this disorder of kidney called?
82
A. Acute renal failure
B. Acute diffuse glomerulonephritis
C. Uremic coma
D. Chronic renal failure
E. Chronic glomerulonephritis

22. Sharp weakness, paleness of skin, lost of consciousness appeared in a patient the next
day after resection of his stomach. The patient’s BP is 70/40 mmHg; pulse is 160 beats
per minute. In the patient’s blood test Hb is 70 g/L, erythrocytes are 2.3*1012/L. What
pathology appeared in a patient?
A. Hemorrhagic collapse
B. Pain shock
C. Traumatic shock
D. Orthostatic collapse
E. Cardiogenic collapse

23. Sharp decrease of systolic BP down to 60 mmHg, tachycardia of 140 beats per
minute, dyspnea, and loss of consciousness developed in a patient on the second day after
myocardial infarction. What pathways have the most important significance in
pathogenesis of this shock?
A. Orthostatic collapse
B. Decrease in blood volume
C. Development of paroxysmal tachycardia
D. Increase in excitability of the myocardium by products of necrotic decay
E. Development of anaphylactic reaction to myocardial proteins

24. A driver was admitted to an emergency department after a car accident. He does not
react to questions; he is indifferent to everything, pale; he has shallow and infrequent
respiration and BP of 75/50 mmHg. Name the principal link in pathogenesis of this
pathology.
A. Inhibition of CNS
B. Excitement of CNS
C. Loss of blood
D. Toxemia
E. Redistribution of blood

25. To compensate considerable loss of blood resulting from knife wounding of liver, a
30-yer-old patient with blood of IV (AB) Rh (-) group was transfused with blood of
IV(AB) Rh (+) group. Requirement in repeated blood transfusion appears in several days.
What kind of blood is it possible to use for transfusion?
A. IV(AB) Rh (-)
B. I(O) Rh (+)
C. II(A) Rh (-)
D. IV(AB) Rh (+)
E. III(B) Rh (-)

25. A victim of a traffic accident was received by the intensive care unit. The patient is in
a grave condition that can be characterized as a severe pathologic process that leads to
83
exhaustion of vital functions and puts the patient into the marginal state between life and
death due to critical reduction of capillary circulation in the affected organs. The patient
is in the state of:
A. Shock
B. Collapse
C. Coma
D. Agony
E. Preagony
F.
26. A patient suffers from posttraumatic hemorrhage that resulted in development of
hemorrhagic shock. What volume of circulating blood was lost by the patient?
A. 25-40%
B. 40-50%
C. 12-25%
D. 50-75%
E. 3-20%

27. A patient on the 2nd day after cardiac infarction presents with acute decrease of
systolic blood pressure down to 60 mm Hg with tachycardia 140/min., dyspnea, loss of
consciousness. What mechanism is essential in the pathogenesis of shock developed in
this case?
A. Decreased cardiac output
B. Increased myocardial excitability caused by products of necrotic disintegration
C. Decreased circulating blood volume
D. Development of paroxysmal tachycardia
E. Development of anaphylactic reaction to myocardial proteins

28. After a traffic accident a man presents with severe blood loss, consciousness di-
sturbance, low blood pressure, as well as compensatory activation of the renin-
angiotensin system, which results in:
A. Hyperproduction of aldosterone
B. Increased blood coagulation
C. Intensification of erythropoiesis
D. Hyperproduction of vasopressin
E. Intensification of heart contractions

29. A 16-year-old girl fainted when she triedtoquicklychangeherpositionfromhorisontal to


vertical. What caused the loss of consciousness in the girl?
A. Decreased venous return
B. Increased venous return
C. Increased central venous pressure
D. Decreased oncotic plasma pressure
E. Increased arterial pressure

30. An accident had resulted in a 65-year-old man drowning in a lake. Resuscitation


measures allowed to restorehisrespirationandcardiacfunction. What factor prolongs the
period of apparent death?
A. Hypothermia
84
B. Hyperthermia
C. Prolonged preagony and agony
D. Elderly age
E.-

31. A 62-year-old patient has been hospitalizedduetomassivecerebralhemorrhage. Blood


pressure is 70/30 mm Hg, heart rate is 120/min., respiratory rate is 4/min., unconscious,
no response to external stimuli.Such condition can be determinedas:
A. Coma
B. Shock
C. Collapse
D. Stress
E. Agony

32. After a road accident a victim has tachycardia, arterial blood pressure 130/90 mm Hg,
tachypnoe, the skin is pale and dry, excitation of central nervous system is observed.
What shock stage is the patient most likelyin?
A. Erectile
B. Terminal
C. Torpid
D. Preshock (compensation stage)
E. Agony

33. Injection of an anaesthetic before the tooth extraction resulted in development of


anaphylactic shock accompanied by oliguria. What pathogenetic mechanism caused the
decrease in diuresis in this case?
A. Decrease in hydrostatic pressure in the renal corpuscle capillaries
B. Increase in hydrostatic pressure in the Bowman’s capsule
C. Damage of the glomerular filter
D. Increase in oncotic pressure of blood plasma
E. Increase in vasopressin secretion

Pathophysiology of the endocrine system

1. Periodic renal colics attacks are observed in a woman with primary


hyperparathyroidizm. Ultrasonic examination revealed small stones in the kidneys. What
is the most plausible reason of the stones formation?
A. Hypercalcemia
B. Hyperphosphatemia
C. Hypercholesterinemia
D. Hyperuricemia
E. Hyperkalemia

2.A 44 year old woman complains of general weakness, heart pain, significant increase of
body weight. Objectively: moon face, hirsutism, AP is 165/100 mm Hg, height - 164 cm,
weight - 103 kg; the fat is mostly accumulated on her neck, thoracic girdle, belly. What is
the main pathogenetic mechanism of obesity?
A. Increased production of glucocorticoids
85
B. Reduced production of thyroid hormones
C. Increased insulin production
D. Reduced glucagon production
E. Increased mineralocorticoid production

3. A girl is diagnosed with adrenogenital syndrome (pseudohermaphroditism). This


pathology was caused by hypersecretion of the following adrenal hormone:
A. Androgen
B. Estrogen
C. Aldosterone
D. Cortisol
E. Adrenalin

4. A concentrated solution of sodium chloride was intravenously injected to an animal.


This caused decreased reabsorption of sodium ions in the renal tubules. It is the result of
the following changes of hormonal secretion:
A. Aldosterone reduction
B. Aldosterone increase
C. Vasopressin reduction
D. Vasopressin increase
E. Reduction of atrial natriuretic factor

5. Rats being under stress have muscular hypertonia and high arterial pressure, high
glucose concentration in blood and intensified secretion of corticotropin and
corticosteroids. In what stress phase are these animals?
A. Antishock phase
B. Exhaustion
C. Shock phase
D. Erectile
E. Terminal

6. A 46-year-old patient suffering from the diffuse toxic goiter underwent resection of the
thyroid gland. After the surgery the patient presents with appetite loss, dyspepsia,
increased neuromuscular excitement. The body weight remained unchanged. Body
temperature is normal. Which of the following has caused such a condition in this
patient?
A. Reduced production of parathormone
B. Increased production of thyroxin
C. Increased production of calcitonin
D. Increased production of thyroliberin
E. Reduced production of thyroxin

7. A patient with android-type obesity had been suffering from arterial hypertension,
hyperglycemia, glycosuria for a long time and died from the cerebral haemorrhage.
Pathologic examination revealed pituitary basophil adenoma, adrenal cortex hyperplasia.
What is the most likely diagnosis?
A. Itsenko-Cushing’s syndrome
B. Diabetes mellitus
86
C. Acromegalia
D. Pituitary nanism
E. Adiposogenital dystrophy

8. Examination of a patient revealed overgrowth of facial bones and soft tissues, tongue
enlargement, wide interdental spaces in the enlarged dental arch. What changes of the
hormonal secretion are the most likely?
A. Hypersecretion of the somatotropic hormone
B. Hyposecretion of the somatotropic hormone
C. Hypersecretion of insulin
D. Hyposecretion of thyroxin
E. Hyposecretion of insulin

9. People adapted to high external temperatures have such peculiarity: profuse sweating
isn’t accompanied by loss of large volumes of sodium chloride. This is caused by the
effect of the following hormone upon the perspiratory glands:
A. Aldosterone
B. Vasopressin
C. Cortisol
D. Tgyroxin
E. Natriuretic

10. A patient suffering from pheochromocytoma complains of thirst, dry mouth, hunger.
Blood test for sugar revealed hyperglycemia. What type of hyperglycemia is it?
A. Adrenal
B. Hypercorticoid
C. Alimentary
D. Somatotropic
E. Hypoinsulinemic

11. Examination of a 42 year old patient revealed a tumour of adenohypophysis.


Objectively: the patient’s weight is 117 kg, he has moon-like hyperemic face, red-blue
striae of skin distension on his belly. Osteoporosis and muscle dystrophy are present. AP
is 210/140 mm Hg. What is the most probable diagnosis?
A. Cushing’s disease
B. Cushing’s syndrome
C. Conn’s disease
D. Diabetes mellitus
E. Essential hypertension

12. The secretion of which hypophysial hormones will be inhibited after taking the oral
contraceptives containing sex hormones?
A. Gonadotropic hormone
B. Vasopressin
C. Thyrotrophic hormone
D. Somatotropic hormone
E. Ocytocin

87
13. Parodontitis is treated with calcium preparations and a hormone that stimulates tooth
mineralization and inhibits tissue resorption. What hormone is it?
A. Calcitonin
B. Parathormone
C. Adrenalin
D. Aldosterone
E. Thyroxine

14. A 20 year old patient complains of morbid thirst and huperdiuresis (up to 10 l daily).
Glucose concentration in blood is normal but it is absent in urine. The patient has been
diagnosed with diabetes insipidus. What hormonal drug is the most appropriate for
management of this disorder?
A. Vasopressin
B. Cortisol
C. Thyroxin
D. Oxytocin
E. Insulin

15. A 5-month-old boy was hospitalized for tonic convulsions. He has a lifetime history
of this disease. Examination revealed coarse hair, thinned and fragile nails, pale and dry
skin. In blood: calcium - 1,5 millimole/l, phosphor - 1,9 millimole/l. These changes are
associated with:
A. Hypoparathyroidism
B. Hyperparathyroidism
C. Hyperaldosteronism
D. Hypoaldosteronism
E. Hypothyroidism

16. A 19-year-old male was found to have an elevated level of potassium in the
secondary urine. These changes might have been caused by the increase in the following
hormone level:
A. Aldosterone
B. Oxytocin
C. Adrenaline
D. Glucagon
E. Testosterone

17. A patient with signs of osteoporosis and urolithiasis has been admitted to the
endocrinology department. Blood test revealed hypercalcemia and hypophosphatemia.
These changes are associated with abnormal synthesis of the following hormone:
A. Parathyroid hormone
B. Calcitonin
C. Cortisol
D. Aldosterone
E. Calcitriol

88
18. A 30-year-old female exhibits signs of virilism (growth of body hair, balding temples,
menstrual disorders). This condition can be caused by the overproduction of the
following hormone:
A. Testosterone
B. Oestriol
C. Relaxin
D. Oxytocin
E. Prolactin

19. A 41-year-old male patient has a history of recurrent attacks of heartbeats


(paroxysms), profuse sweating, headach. Examination revealed hypertension,
hyperglycemia, increased basal metabolic rate, and tachycardia. These clinical
presentations are typical for the following adrenal pathology:
A. Hyperfunction of the medulla
B. Hypofunction of the medulla
C. Hyperfunction of the adrenal cortex
D. Hypofunction of the adrenal cortex
E. Primary aldosteronism

20. In the course of an experiment adenohypophysis of an animal has been removed. The
resulting atrophy of thyroid gland and adrenal cortex has been caused by deficiency of
the following hormone:
A. Tropic hormones
B. Thyroid hormones
C. Somatotropin
D. Cortisol
E. Thyroxin

21.A 40-year-old woman on examination presents with intensified basal metabolic rate.
What hormone present in excess leads to such condition?
A. Triiodothyronine
B. Thyrocalcitonin
C. Glucagon
D. Aldosterone
E. Somatostatin

22. After a case of sepsis a 27-year-old woman developed ”bronzed” skin discoloration
characteristic of Addison’s disease. Hyperpigmentation mechanism in this case is based
on increased secretion of:
A. Melanocyte-stimulating hormone
B. Somatotropin
C. Gonadotropin
D. β-lipotropin
E. Thyroid-stimulating hormone

23. A 16-year-old girl presents with no hair on the pubis and in the armpits, her
mammary glands are underdeveloped, no menstruations. What hormone imbalance can it
be indicative of?
89
A. Ovarian failure
B. Hyperthyroidism
C. Hypothyroidism
D. Pancreatic islet failure
E. Adrenal medulla hyperfunction
24. A patient demonstrates sharp decrease of pulmonary surfactant activity. This condi-
tion can result in:
A. Alveolar tendency to recede
B. Decreased airways resistance
C. Decreased work of expiratory muscles
D. Increased pulmonary ventilation
E. Hyperoxemia
25. On examination the patient presents with hirsutism, moon-shaped face, stretch marks
on the abdomen. BP is 190/100 mm Hg, blood glucose is 17.6 mmol/L. What pathology
is such clinical presentation characteristic of?
A. Adrenocortical hyperfunction
B. Hyperthyroidism
C. Hypothyroidism
D. Gonadal hypofunction
E. Hyperfunction of the insular apparatus

26. A 19-year-old young man was examined in the nephrology clinic. High calcium was
detected in his secondary urine. What hormone is likely to cause such change, if it is
produced in excess?
A. Aldosterone
B. Oxytocin
C. Adrenaline
D. Glucagon
E. Testosterone

27. A patient has undergone surgical removal of the pylorus. Decreased secretion of the
following hormone can be expected:
A. Gastrin
B. Histamine
C. Secretin
D. Cholecystokinin
E. Gastric inhibitory polypeptide

28. A 30-year-old woman complains of intense thirst and dryness of the mouth that
developed after a severe emotional shock. Laboratory analysis revealed increase of the
patient’s blood sugar level up to 10 mmol/L. What endocrine gland is affected in the
patient?
A. Pancreas
B. Thyroid gland
C. Gonads
D. Adrenal glands
E. Pineal gland

90
29. Due to trauma the patient’s parathyroid glands have been removed, which resulted in
inertness, thirst, sharp increase of neuromuscular excitability. Metabolism of the
following substance is disturbed:
A. Calcium
B. Manganese
C. Chlorine
D. Molybdenum
E. Zinc

30. A patient presents with high content of vasopressin (antidiuretic hormone) in the
blood. What changes in the patient’s diuresis will occur?
A. Oliguria
B. Polyuria
C. Anuria
D. Glycosuria
E. Natriuria

31. A patient presents with osteoporosis; hypercalcemia and hypophosphatemia are


observed in the patient’s blood. What is the cause of this condition?
A. Increased parathormone secretion
B. Increased thyroxin secretion
C. Inhibited parathormone secretion
D. Increased corticosteroid secretion
E. Inhibited corticosteroid secretion

32. After introduction of adrenaline the patient’s blood glucose level increased. It is
caused by intensified:
A. Glycogenolysis in the liver
B. Glycolysis in the liver
C. Glycolysis in the skeletal muscles
D. Glycogen synthesis
E. Glycogenolysis in the muscles

33. During removal of the hyperplastic thyroid gland of a 47-year-old woman, the
parathyroid gland was damaged. One month after the surgery the patient developed signs
of hypoparathyroidism: frequent convulsions, hyperreflexia, laryngospasm.What is the
most likely cause of the patient’s condition?
A. Hypocalcemia
B. Hyponatremia
C. Hyperchlorhydria
D. Hypophosphatemia
E. Hyperkalemia

34. On examination the patient presents with hirsutism, moon-shaped face, stretch marks
on the abdomen. BP is 190/100 mm Hg, blood glucose is 17,6 mmol/l. What pathology is
such clinical presentation characteristic of?
A. Adrenocortical hyperfunction
B. Hyperthyroidism
91
C. Hypothyroidism
D. Gonadal hypofunction
E. Hyperfunction of the insular apparatus
35.A30-year-old woman developed the signs of virilism (body hair growth, balding
temples, disturbed menstrual cycle). What hormone can cause this condition when
hyperproduced?
A. Testosterone
B. Estriol
C. Relaxin
D. Oxytocin
E. Prolactin

36. After a severe stress the patient presents with eosinopenia in the blood test. In this
case the decreased number of eosinophils can explain changes in the level of the
followinghormones:
A. Glucocorticoids
B. Adrenaline
C. Insulin
D. Mineralocorticoids
E. Vasopressin

37. Corticosteroid hormones regulate the adaptation processes of the body as a whole to
environmental changes and ensure the maintenance of internal homeostasis. What
hormone activates the hypothalamopituitary-adrenal axis?
A. Corticoliberin
B. Somatoliberin
C. Somatostatin
D. Corticostatin
E. Thyroliberin

38. On examination the patient is found to have low production of adrenocorticotropic


hormone.How would this affect production of the other hormones?
A. Decrease adrenocorticotropic hormones synthesis
B. Decrease hormone synthesis in the adrenal medulla
C. Decrease insulinsynthesis
D. Increase sex hormones synthesis
E. Increase thyroid hormones synthesis

39. A 56-year-old man presents with parathyroid tumor. The following is observed:
muscle weakness, osteoporosis, bone deformation, nephroliths consisting of oxalates and
phosphates. The patient’s condition is caused by:
A. Increased secretion of parathyroid hormone
B. Decreased secretion of parathyroid hormone
C. Increased secretion of calcitonin
D. Decreased secretion of calcitriol
E. Increased secretion of thyroxin

92
40. A doctor has established significant growth retardation, disproportional body build,
and mental deficiency of a child. What is the most likely cause of this pathology?
A. Hypothyroidism
B. Insufficient nutrition
C. Hyperthyroidism
D. Genetic defects
E. Hypopituitarism

41. Due to morbid affection of the supraoptic and paraventricular nuclei of the
hypothalamus a 40-year-old patient has developed polyuria (10-12 liters per day) and
polydipsia. The following hormone is deficient, thus leading to this disturbance:
A. Vasopressin
B. Oxytocin
C. Corticotropin
D. Somatotropin
E. Thyrotropin

42. A patient is in the state of hypoglycemic coma. What hormone can cause this
condition if overdosed?
A. Insulin
B. Progesterone
C. Cortisol
D. Somatotropin
E. Corticotropin

43. A 30-year-old woman exhibits signs of virilism (growth of body hair, balding
temples, menstrual disorders). This conditioncanbecausedbyoverproductionofthe
followinghormone:
A. Testosterone
B. Oestriol
C. Relaxin
D. Oxytocin
E. Prolactin

44. A patient with signs of osteoporosis and urolithiasis has been admitted to an
endocrinology department. Blood test revealed hypercalcemia and hypophosphatemia.
These changes are associated with abnormal synthesis of the followinghormone:
A. Parathyroid hormone
B. Calcitonin
C. Cortisol
D. Aldosterone
E. Calcitriol

45. Leading symptoms of primary hyperparathyroidism are osteoporosis and renal


damage resulting in urolithiasis development.Whatsubstancesarethebasis of uroliths in
such cases?
A. Calcium phosphate
B. Uric acid
93
C. Cystine
D. Bilirubin
E. Cholesterol
46. For people adapted to high external temperatures profuse sweating is not
accompanied by loss of large volumes of sodiumchloride.Thisiscausedbytheeffect the
following hormone has on perspiratory glands:
A. Aldosterone
B. Vasopressin
C. Cortisol
D. Tgyroxin
E. Natriuretic

47. A 41-year-old man has a history of recurrent attacks of heartbeats


(paroxysms), profuse sweating, headaches. Examination revealed hypertension,
hyperglycemia, increased basal metabolic rate, and tachycardia. These clinical
presentations are typical of the following adrenalpathology:
A. Hyperfunction of the medulla
B. Hypofunction of the medulla
C. Hyperfunction of the adrenal cortex
D. Hypofunction of the adrenal cortex
E. Primary aldosteronism

48. A 40-year-old woman suffering from diffuse toxic goiter presents with constant
increase of her body temperature. What mechanism results in such clinical presentation?
A. Separation of oxidation and phosphorization in cell mitochondria
B. Increased breakdown of glycogen in hepatic cells
C. Increased catabolism of protein in cells
D. Increased excitability of nerve cells
E. Increased cell sensitivity to catecholamines

49. A 19-year-old young man has been examined in a nephrological hospital. Increased
potassium content was detected in secondary urine of the patient.Such changes have been
mostlikely caused by the increased secretion of the following hormone:
A. Aldosterone
B. Oxytocin
C. Adrenalin
D. Glucagon
E.Testosterone

50. A 43-year-old woman complains of weight loss, hyperhidrosis, low-grade fever,


increased irritability. She has been found to have hyperfunction of the sympathetic-
adrenal system and basal metabolism. These disorders can be caused by hypersecretion of
thefollowing hormone:
A. Thyroxine
B. Somatotropin
C. Corticotropin
D. Insulin
E. Aldosterone
94
51. A 49-year-old patient was found to have a disproportionate enlargement of hands,
feet, nose, ears, superciliary arches and cheek bones. Blood test revealed hyperglycemia,
impaired glucose tolerance. What is the most likely cause of this pathology development?
A. Hypersecretion of growth hormone
B. Posterior pituitary hormone hypersecretion
C. Insulin hyposecretion
D. Vasopressin hyposecretion
E. Glucocorticoid hypersecretion

Pathophysiology of the nervous system

1. A 60-year-old patient was diagnosed with hypothalamic lateral nuclei stroke. What
changes in patient’s behavior may be expected?
A. The rejection of food
B. Aggressive behaviour
C. Depression
D. Thirst
E. Unsatisfied hunger

2. A patient with disturbed cerebral circulation has problems with deglutition. What part
of brain was damaged?
A. Brainstem
B. Cervical part of spinal cord
C. Forebrain
D. Interbrain
E. Midbrain

3. A 28 year old man had a gunshot wound of shin that resulted in an ulcer from the side
of the injury. What is the main factor of neurodystrophy pathogenesis in this case?
A. Traumatization of peripheral nerve
B. Psychical stress
C. Microcirculation disturbance
D. Infection
E. Tissue damage

4. A 35 year old man got an injury that caused complete disruption of spinal cord at the
level of the first cervical segment. What respiration changes will be observed?
A. It will come to a standstill
B. No changes will be observed
C. Diaphragmal respiration will be maintained, thoracic respiration will disappear
D. Thoracic respiration will be maintained, diaphragmal respiration will disappear
E. It will become infrequent and deep

5. An experimental animal has lost orientative reflexes as a result of destruction of certain


brainstem structures. What structures had been destroyed?
A. Quadrigeminal plate
B. Medial nuclei of the reticular formation
95
C. Red nuclei
D. Vestibular nuclei
E. Black substance

6. A patient has corestenoma. What is the reason of such condition?


A. Increased tonus of parasympathetic centres
B. Increased tonus of sympathetic centres
C. Increased activity of sympathoadrenal system
D. Adrenaline action
E. Noradrenaline action

7. A patient underwent an extraction of a part of a CNS structures by medical indications.


As a result of the extraction the patient developed atony, astasia, intention tremor, ataxy
and adiadochokinesis. Which part of CNS structure had been extracted?
A. Cerebellum
B. Amygdaloid corpus
C. Hippocamp
D. Basal ganglions
E. Limbic system

8. After a long training session a sportsman has developed fatigue accompanied by abrupt
performance decrement. What link of the reflex arch was the fatigue initiated in?
A. Nerve centres
B. Afferent conductor
C. Receptors
D. Efferent conductor
E. Muscles

9. After the traumatic tooth extraction a patient is complaining of acute, dull, poorly-
localized pain in gingiva, body temperature rise up to 37, 5oC. The patient has been
diagnosed with alveolitis. Specify the kind of pain in this patient:
A. Protopathic
B. Epicritic
C. Visceral
D. Heterotopic
E. Phantom

10. As a result of a trauma a patient has damaged anterior roots of spinal cord. What
structures have been affected?
A. Axons of motoneurons and axons of neurons of lateral horns
B. Central processes of sensitive neurons of spinal ganglions
C. Peripheral processes of sensitive spinal ganglions
D. Axons of neurons of lateral horns
E. Dendrites of neurons of spinal ganglions

11. One of sections of central nervous system has layerwise arrangement of neurocytes.
Among them there are cells of the following forms: stellate, fusiform, horizontal,
pyramidal. What section of central nervous system is this structure typical for?
96
A. Cortex of cerebrum
B. Spinal cord
C. Cerebellum
D. Medulla oblongata
E. Hypothalamus

12. It has been experimentally proven that the excitation of the motor neurons of flexor
muscles is accompanied by the inhibition of the motor neurons of extensor muscles.
What type of inhibition underlies this phenomenon?
A. Reciprocal
B. Inhibition after excitation
C. Pessimal
D. Feedback
E. Lateral

13. A patient got a gunshot wound of hip which damaged the sciatic nerve. Any impact
on the affected limb causes severe, excruciating pain. What mechanism of pain is most
likely in this case?
A. Causalgic
B. Reflex
C. Phantom
D. Endorphin hypofunction
E. Enkephalin hypofunction

14. A male with a lesion of one of the CNS parts has asthenia, muscular dystonia, balance
disorder. Which CNS part has been affected?
A. Cerebellum
B. Black substance
C. Reticular formation
D. Red nuclei
E. Vestibular nuclei

15. As a result of a cranio-cerebral injury a patient has a decreased skin sensitivity. What
area of the cerebral cortex may be damaged?
A. Posterior central gyrus
B. Occipital region
C. Cingulate gyrus
D. Frontal cortex
E. Anterior central gyrus

16. An animal has an increased tonus of extensor muscles. This is the result of enhanced
information transmission to the motoneurons of the spinal cord through the following
descending pathways:
A. Vestibulospinal
B. Medial corticospinal
C. Reticulospinal
D. Rubrospinal
E. Lateral corticospinal
97
17. A patient complains that at the bare mention of the tragic events that once occurred in
his life he experiences tachycardia, dyspnea and an abrupt rise in blood pressure. What
structures of the CNS are responsible for these cardiorespiratory reactions in this patient?
A. Cerebral cortex
B. Cerebellum
C. Lateral hypothalamic nuclei
D. Specific thalamic nuclei
E. Quadrigemina of mesencephalon

18. A patient complaining of pain in the left shoulder-blade region has been diagnosed
with miocardial infarction. What kind of pain does the patient have?
A. Radiating
B. Visceral
C. Phantom
D. Protopathic
E. Epicritic

19. During an animal experiment, surgical damage of certain brain structures has caused
deep prolonged sleep. What structure is most likely to cause such condition, if damaged?
A. Reticular formation
B. Basal ganglion
C. Red nuclei
D. Hippocampus
E. Cerebral cortex

20. The right leg of a 40-year-old woman measured at the shin level is by 2 cm smaller in
the diameter than the left leg. Ankle- jerk (Achilles) and knee-jerk reflexes are absent on
the right. What is the most likely mechanism of hyporeflexia development during
peripheral paralysis?
A. Disturbed conduction of stimulation
B. Inhibition of pyramidal motoneuron
C. Disturbed synaptic impulse transmission
D. Activation of excitatory impulses from the CNS
E. Disturbed perception of stimulation

21. A person becomes less receptive to pain in physically or emotionally straining


situations due to activation of:
A. Antinociceptive system
B. Thyroid gland functions
C. Nociceptive system
D. Adrenal glands functions
E. Parasympathetic nervous system

22. An 84-year-old patient suffers from parkinsonism. One of the pathogenetic


development elements of this disease is deficiency of a certain mediator in some of the
brain structures. Name this mediator:
A. Dopamine
98
B. Adrenaline
C. Noradrenaline
D. Histamine
E. Acetylcholine

23. A patient with hypertension has developed headache, tinnitus, vomiting, high BP up
to 220/160 mm Hg. On examination: facial asymmetry on the right, volitional mobility is
absent, increased tendon reflexes and muscle tone of extremities on the right. What motor
disorder of nervous system occurred in this case?
A. Hemiplegia
B. Paraplegia
C. Tetraplegia
D. Hyperkinesis
E. Monoplegia

24. After the traumatic tooth extraction a patient is complaining of severe dull poorly-
localized pain in gingiva, body temperature riseup to 37,5oC. The patient has been
diagnosed with alveolitis. Specify the kind of pain in this patient:
A. Protopathic
B. Epicritic
C. Visceral
D. Heterotopic
E. Phantom

25. Parkinson’s disease is caused by disrupteddopaminesynthesis.Whatbrain structure


synthesizes this neurotransmitter?
A. Substantia nigra
B. Pallidum
C. Quadrigeminal plate
D. Red nuclei
E. Hypothalamus

99

You might also like